UWorld OBGYN

Réussis tes devoirs et examens dès maintenant avec Quizwiz!

What groups of women require endometrial biopsy; If biopsy shows: 1- Benign appearing endometrial cells 2- Atypical glandular cells 3- Atypical glandular cells, favor neoplastic

1- Benign appearing Post-menopausal women >/= 45ys old w/: - Abnormal uterine bleeding - Risk for endometrial hyperplasia 2- Atypical glandular cells - Women > 35 - women at risk for endometrial hyperplasia 3- Atypical glandular cells, favor neoplastic - All women

What vaccinations are indicated for High risk pts only in pregnancy?

1- Hep B 2- Hep A 3- Pneumococcus 4- H.influenzae 5- Meningococcus 6- VZV immunoglobulins

What vaccination are CI in pregnancy?

1- MMR 2- HPV 3- Live attenuated influenza 4- Varicella

How do you score a BPP? 1- NST 2- Amniotic fluid Volume 3- Fetal Mvmts 4- Fetal tone 5- Fetal breathing mvmts

1- NST Reactive FHR monitoring 2- Amniotic fluid Volume Single pocket >/= 2cm x 1cm OR Amniotic fluid index >5 3- Fetal Mvmts >/=3 general body mvmts 4- Fetal tone >/=1 episode of flexion/extension of fetal limbs or spine 5- Fetal breathing mvmts >/=1 breathing episode for >/= 30 sec

A 26F, G1P0, Presents for a prenatal visit. She did not keep accurate records of her menstrual cycle but the last one was approximately 7 months ago. Early in pregnancy, an US showed that she was approximately 6 wks, giving her a current gestational age of 32 wks. The pt's BP is 110/60, pulse is 90/min. On exam, her fundal ht is 26 cm. Fetal heart tones are 140/min. An US calculates the age to be 28 wks based on fetal biparietal diameter & femur length. The measured fetal AC is consistent w/ a gestational age of 26 wks. Which of the following is the most accurate method for estimating the current gestational age? (First trimester US OR Third trimester US)

1st trimester US > 1st trimester US + Crown rump length measurement is the most accurate method of determining gestational age >> *becomes less accurate as the pregnancy progresses* >> as there is minimal variability in size amount fetuses during the first trimester

A 15F, is brought in by mom d/t concerns that her daughter has not had a menstrual period. The pt was born at 36 wks but has had no developmental delay. She is doing well in school & plays on the tennis team. The pt is healthy & takes no meds. She is not sexually active & does not use tobacco, alcohol, or drugs. FHx is sig for a maternal aunt w/ primary infertility & a maternal grandmother w/ ovarian cancer. Ht is 174 cm (5'7") & wt 63kg (138.9lb). Vitals are normal. There is no acne or excessive hair growth. Breast development is Tanner stage 5. There is scant axillary & pubic hair. On Pelvic exam, the external genitalia appear normal & speculum exam shows a blind vaginal pouch. The uterus, cervix, & ovaries are absent on bimanual exam. Karyotype analysis of this pt is most likely going to show which of the following? (46, XY OR 47 XXY)

46, XY - Pt has AIS >> Pt has male karyotype but female phenotype >> Presentation includes 1- *Scant axillary & pubic hair w/ small vagina* (missing upper 1/3 of vagina) 2- *would be missing uterus, cervix, & ovaries* 47, XXY > Kleinfelters >> would present as male phenotype but with small, undescended testis

A 13f, presents for eval of acne. She began to develop acne over her forehead & chin 4 months ago. She used OTC topical salicylic acid, but the acne did not improve. Now, the pt has painful acne over her face & upper chest. She participates in a soccer league after school, & her mother feels that her exercise regimen is too strenuous & stressful & contributes to her acne. The pt has no sig PMH & had had no surgeries. She has not had a menstrual period. The girl has grown 10.1cm (4in) in the last year; her ht & wt are at the 70th percentile for age. Vitals are normal. Nodulocystic acne is present across the face & upper chest. There is no breast bud development. The abd is nontender & has no palpable masses. External pelvic exam shows the clitoris protruding from the clitoral hood & bilateral masses in the labia majora. Which of the following is the most likely Dx? (5-alpha reductase OR AIS)

5- alpha reductase > Pt are unable to make DHT which is responsible for the development of male external genitalia (penis/prostate) >> pts with 5-alpha reductase would have *no male external genitalia* - Would also have undescended testis *Bilateral labial masses* - Appear phenotypically female at birth but *no breast development* as they go thru puberty AIS > Pts would also not develop ext male genitalia >> and would be phenotypically female at birth > Can also have undescended testis >> *pt would have Breast development* at puberty b/c the defective androgen receptor is unable to inhibit breast tissue proliferation

A 16y/o F, presents d/t not having started her menstrual period. The pt says that all her friends have started their periods & she is concerned that she is "behind". She has had no nipple discharge of change in vision or wt. The pt has no chronic medical conditions & takes no meds. She recently became sexually active & uses condoms. Ht is at 70th percentile & wt is at 40th percentile for age. Vitals are normal. On PE, there is sexual maturity rating (tanner) stage 4 Breast development w/ minimal axillary & pubic hair. The external genitalia are normal & the vagina is 3cm in length; the cervix is not visible. No uterus, cervix, or ovaries are palpated on bimanual exam. Labs include a negative pregnancy test & testosterone level of 400 ng/dL (N: 15-75 F) Which of the following is the most likely Dx? (5-alpha reductase deficiency OR AIS)

AIS > Pt w/ primary amenorrhea + male range testosterone levels most likely has AIS >> pts are genotypically male (46XY) but phenotypically female d/t nonfunctioning androgen receptor >> *Think breast development + high testosterone levels* Clinical features - Phenotypically female - *Absent/scant axillary & pubic hair & short vagina* - Female external genitalia - *Absent uterus, cervix, & upper 1/3 of vagina* - High testosterone levels 5-alpha reductase deficiency > Appear phenotypically female at birth >> *develop virilization (clitoromegaly) & have no breast development*

A 53F, gravida 2 para 2, presents d/t right-sided pelvic pain that has worsened over the past 3 months. She has experienced bloating & hot flashes since her last menstrual period a year ago. The pt has recently become sexually active w/ a new partner & is not using condoms. She was Dx'd & treated for chlamydia in her 4-'s. She has no other medical problems. She had a bilateral tubal ligation at age 35 after CS of her second child. The pt smokes 10 ciggs daily & drinks alcohol socially. Temp is 98F, BP is 110/70. Leukocyte count is 8200. B-hCG is undetectable. Pelvic US shows a 7cm right ovarian mass w/ solid components, thick separations, & a moderate amount of peritoneal fluid. Which of the following is the most likely explanation for these findings? (Abnormal proliferation of tubal epithelium OR Excessive growth of well differentiated ectodermal cells OR Obstruction from fluid accumulation in the Fallopian tube)

Abnormal proliferation of tubal epithelium > *Epithelial ovarian carcinoma* involves the ovary, fallopian tube, and peritoneum >US would reveal a large ovarian mass w/ *thick septations, solid components, a& peritoneal free fluid (ascites)* > symptoms would be - Bloating, pelvic pain - Early satiety/anorexia - Abd distention > More common in older women Excessive growth of well differentiated ectodermal cells > US would show a *homogenous cyst w/ internal echoes (ground glass)* Obstruction from fluid accumulation in the Fallopian tube > AKA *hydrosalpinx* >> US would show a mass separate from the ovary rather than an ovarian tumor > Would not be associated w/ malignant mass

A 36F, gravida 2 para 1, at 35 wks comes to L & D for vaginal bleeding & constant back pain. The pt has a Hx of HTN that has been well controlled w/ labetalol. Her prenatal labs & US were normal. Temp is 98.4F, BP 104/70, Pulse 118/min. PE shows a smooth, firm & distended uterus w/ a fundal ht of 38cm. Speculum exam shows a small amount of bleeding & a closed cervix. The first 10 mins of fetal monitoring shows a baseline HR of 160/min, minimal variability, and 3 late decels. Uterine contractions are observed every 1-2 mins. Which of the following is the most likely Dx? (Abruptio placentae OR Placenta previa OR Preterm labor)

Abruptio placentae > Typically presents w/ - *Abd and/or Constant back pain + Bleeding* - *tachy* in pt - Vaginal bleeding >> which may cause: ------*frequent contractions* blood may have an uterotonic effect, causing a firm uterus & unusually low-amplitude but ------*Abnormal fetal HR tracing* ------*Distended uterus* (d/t concealed bleeding) Placenta Previa > *Painless* vaginal bleeding (pt has constant pain) Preterm labor > Pt would have contractions which would present w/ *pain that is intermittent* >> this pt has constant pain > would not have distended uterus

A 33F, G2P2 presents for eval of abnormal uterine bleeding. Ten months ago, the pt had a spontaneous vaginal delivery complicated bu a postpartum hemorrhage that required a blood transfusion & emergent suction curettage. Since the delivery, the pt has had increasingly irregular menses & has been amenorrheic for the past 3 months. She is bottle-feeding her infant & is not taking contraceptives. The pt lost 30 lbs since the delivery & is now below her pre-pregnancy wt. BMI is 22, BP is 110/60, pulse is 62/min. Visual fields are intact. Thyroid has no palpable masses. Cardiopulmonary exam is unremarkable. PE shows a minimally rugated vagina; the uterus & cervix are small & non-tender to palpation. Urine pregnancy test is negative. Serum TSH & prolactin levels are normal; FSH is elevated. Pelvic US shows a uterus w/ a thin endometrium & no adnexal masses. Which of the following is the most likely cause of this pt's presentation? (Accelerated ovarian follicle depletion OR Infarction of the pituitary gland)

Accelerated ovarian follicle depletion > Pt presents w/ secondary amenorrhea >> Hypogonadotropic hypogonadism >> *primary ovarian insufficiency* >> which presents w/: 1- Amenorrhea at age < 40 yrs 2- Hypoestrogenic symptoms (hot flashes, etc,) labs: 1- *increased FSH* 2- *Decreased estrogen* (minimally rugated vagina) Physiology > POI affects the ovarian primordial follicles >> *accelerated ovarian primordial follicle depletion* results in lack of oocytes >> causing anovulation >> & decreased estrogen Infarction of pituitary gland > *Sheehans syndrome* >> can occur after *postpartum hemorrhage that results in hypovolemic shock* >> presents w/: 1- Inability or breastfeed *low prolactin* 2- Amenorrhea *d/t Low FSH* Pt has high FSH

A 31F, gravida 1 para 0, presents for a routine prenatal visit at 28 wks. Her initial prenatal visit was 2 wks ago. The pt reports fatigue but is otherwise feeling well. She has no contractions, vaginal bleeding, or leakage of fluid. Fetal mvmt is normal. The pt had a vaginal yeast infection 6 wks ago that she treated w/ an OTC suppository. Her only other med is a prenatal vitamin. The pt does not use tobacco, alcohol, drugs. Temp is 99F, BP is 100/70, BMI is 28. Fetal HR is 155/min. Fundal Ht is appropriate for gestational age. A 1-hr 50g oral GCT shows a blood glucose of 120mg/dL. A urine culture grew >100K colony forming units of E.coli. If left untreated, this pt is at risk for which of the following complications? (Acute pyelonephritis OR Chorioamnionitis)

Acute pyelonephritis > Pt has asymptomatic bacteriuria (growth of > 100K colony forming units in a pt who has no UTI >> in pregnancy increased progesterone levels cause smooth muscle relaxation and ureteral dilation >> leaving pt at increased risk for ascending infection > *E.coli is most common* Chorioamnionitis > d/t the ascent of normal vaginal flora *Polymicrobial infection* into the uterus >> RF include: 1- Prolonged membrane rupture 2- Operative vaginal delivery 3- Polymicrobial

A 28F, G3 P2, at 30 wks presents to the ER w/vaginal bleeding after a MVA. The pt was restrained by a lap & shoulder belt, but the steering wheel struck her abdomen. Her underwear & pants were soaked w/ blood, but she is "not sure is my water broke." The pt's prenatal course has been uncomplicated, as were her prior pregnancies. BP is 95/65, Pulse is 116/min, RR are 22/min, Pulse Ox is 98% on room air. PE shows an alert but anxious-appearing woman w/ abd tenderness & cool extremities. On Speculum, 150mL of blood is evacuated from the vagina & then minimal active bleeding from the cervix is observed. Labs show: - Hb-------------7.6 - Blood type----O+ Fetal Heart monitoring shows contractions every 5 mins; there are no accelerations or decels. Which of the following is the most appropriate next step in MGMT? (Administer IV fluids OR Perform BPP)

Administer IV fluids > Pt likely has abruptio placentae >> Which is presents w/; 1- usually in MVA 2- Hemorrhage 3- Abdominal/ uterine tenderness >> First step in MGMT is *aggressive fluid resuscitation w/ crystalloids & place in lateral decubitus position* BPP > the fetus is relatively protected from maternal trauma >> *morbidity is typically secondary to insults in maternal circulation*

A 32F, primigravida, presents at 28 wks d/t painful contractions that began 2 hours ago & are currently occurring every 5 mins. The pt was last seen in the office a few days ago for routine prenatal care during which she reported symptoms of pelvic pressure. Exam at that time showed a closed cervix. The pt reports frequent fetal mvmts & no leakage of fluid or vaginal bleeding. Temp is 98.7F, BP 125/60, pulse 80/min. A digital exam shows the cervix is 3 cm dilated & 90% effaced w/ a bulging bag. An US confirms vertex presentation. The fetal HR tracing is normal & the tachometer shows uterine contractions every 5 mins. Betamethasone & indomethacin are administered. Which of the following is the most appropriate next step in MGMT? (Administer magnesium sulfate OR Observe & reexamine the cervix in 2 hs OR Test for fetal fibronectin)

Administer magnesium sulfate >Pt is in preterm labor (<37 wks) >> MGMT includes 1- Corticosteroids 2- Tocolytics (<32 wks indomethacin) 3- Magnesium Sulfate (fetal neural protection) Observe & reexamine the cervix > B/c this pt in in preterm labor >> MGMT is focused on decreasing neonatal mortality (if pt was >37 wks a repeat cervical exam in 2hs would be correct Test for fetal fibronectin >Test not needed since we know she is in preterm labor from presentation (Fibronectin is used as RF for preterm labor)

A 36F, G3P2, at 15 wks presents for a routine prenatal visit. The pt has no vaginal bleeding, leakage of fluid, or cramping. She has chronic HTN controlled by 1 med. She is taking daily prenatal vitamin & does not smoke, drink, or use drugs. Her 2 prior pregnancies were uncomplicated. BP is 146/90, pulse is 88/min. BMI is 28. Fetal heart rate is normal. Urinalysis is negative for protein. The first-trimester combined results are reviewed w/ the pt & include an abnormally elevated B-hCG level as well as a 13 wk US that shows an increased nuchal thickness but no other abnormalities. Which of the following is the most appropriate next step in MGMT? (Amniocentesis OR Chorionic villus sampling)

Amniocentesis > At 15 wks the best test that allows for fetal karyotyping is amniocentesis Chorionic villus sampling > *performed at 10-13 wks*

A 22F, G2P1, presents for a routine prenatal visit. She is 28 wks pregnant, feeling well, & gaining wt appropriately. She has no bleeding, fluid leakage, or uterine contractions & feel fetal mvmt. The pt recently ended her relationship w/ the father of the baby. Her first trimester prenatal testing results were as follows: - Rubella------non-immune Which of the following is the most appropriate next step at this gestational age in pregnancy? (Anti-D immune globulin OR urine culture )

Anti-D immune globulin > *Done at 28 -32 wks* also: 1- < 72 hrs after delivery of Rh+ infant 2- < 72 hrs after spontaneous abortion 3- Ectopic pregnancy 4- Threatened abortion 5- Hydatidiform mole 6- Chorionic villus sampling/ Amnio 7- Abd trauma 8- 2nd & 3rd trimester bleeding 9- ECV Urine culture > Screened in 1st trimester

A 18F, Presents after fracturing her distal radius when she fell off a chair. The pt had ambiguous ext genitalia noted at birth, & laparotomy performed at 17 months revealed a normal uterus & Fallopian tubes. Ovarian biopsy performed at the time revealed normal appearing primordial follicles. She has never had a menstrual cycle. BP is 120/78, Ht is 5ft 3in. The pt has nodulocystic acne over the chest & back. No breast development, normal pubic & axially hair, & marked clitoromegaly are present. Labs show a normal female karyotype & normal glucose & serum electrolytes. Estradiol & estrone are undetectable in serum. Serum FSH, LH, Testosterone, and androstenedione conc are high. Pelvic imaging reveals multiple ovarian cysts. Which of the following is the most likely Dx? (Aromatase deficiency OR CAH)

Aromatase deficiency Would present w/: 1- Normal internal genitalia 2- External virilization (clitoromegaly) 3- *Undetectable serum estrogen levels* At birth >> females have normal internal genitalia & ambiguous ext >> d/t high levels of gestational androgens In adolescence >> Pts have > Delayed puberty, > *osteoporosis*, > *Undetectable estrogen levels* (leads to no breast development) > High conc. of gonadotropins that result in polycystic ovaries CAH >Would present w/ normal internal genitalia & ambiguous ext genitalia (adulthood only) *Ambiguous genitalia would NOT BE SEEN at birth* > However, the also have *electrolyte abnormalities - hyponatremia*

A 32F, presents for eval of absent menses. The pt had a vaginal delivery 4 months ago, & she has not had a menstrual period since delivery. Her postpartum course was complicated by a postpartum hemorrhage, requiring blood transfusion & emergency suction & sharp curettage. At her postpartum visit 2 months ago, the pt was started on comb OCP & has had no vaginal bleeding or spotting during her week of placebo pills. Prior to this pregnancy, she had regular monthly menstrual cycles w/ 3-4 days of moderate bleeding. The pt has had increased fatigue since returning to work & is bottle feeding. She has no headaches, galactorrhea, or hot flushes. Vitals are normal. BMI is 31. PE shows clear vaginal discharge throughout the vault & a well-rugated vagina. The uterus is small & anteverted & there are bilateral small, nontender ovaries. Urine pregnancy test is negative. FSH & TSH are normal. Which of the following is the most likely Dx? (Asherman syndrome OR Sheehan syndrome)

Asherman syndrome > Pt had a complicated surgery which required emergency suction & sharp curettage >> These are RF's for asherman syndrome > Pt would *no withdrawal bleeding* during the placebo week of OCP's > *pt would have normal FSH & TSH levels Sheehans > Pt would have had massive hemorrhage requiring blood transfusion & emergency suction curettage >> pt can present w/ 1- Amenorrhea 2- Fatigue 3- Inability to breast feed 4- *low FSH and TSH levels* >> d/t to panhypopituitarism which is not seen in pt

A 26F, presents to the ED d/t a bump on her vaginal Introitus. The pt first noticed the mass 2 days ago, but it has increased in size & now causes discomfort when she walks or exercises. She has had unprotected sex w/ a new partner 6 wks ago & has had 10 lifetime partners. The pt had an abnormal pap last year but had a normal colposcopy. Temp is 98.1, BP is 110/60, pulse is 80/min. On pelvic exam, the pubic hair is shaved. There is a 4cm mobile, soft, nontender cystic mass behind the right posterior labium majus that extends into the vagina. The remainder of the exam is normal. Which of the following is the most likely Dx? (Bartholin duct cyst OR Bartholin gland abscess)

Bartholin duct cyst > Bartholin glands are not palpable unless ductal blockage occurs >> may be asymptomatic >> larger cysts may cause increased tension & friction resulting in *vaginal pressure & discomfort w/ sexual activity, walking, or sitting* - On exam would feel a soft, mobile, *nontender, cystic mass* palpated behind the posterior labium majus Bartholin gland abscess > Pt would have localized erythema & induration & a *tender, fluctuant mass* (Not seen in pt)

A 22F, presents for a routine exam. The pt is well & has no concerns. She is a gymnast, & her main activity is the balance beam. 3months ago, she sustained a vulvar contusion during a competition but has otherwise been healthy. The pt is sexually acitve w/ 2 partners & uses intrauterine devices for contraception. She does not drink, smoke, or use drugs. Exam shows a mobile, soft, nontender, flesh-colored 2-cm mass at the 4 o'cock position at the base of the left labium majus. Which of the following is the most likely Dx? (Bartholin duct cyst OR Condylomata acuminata)

Bartholin duct cyst > Pt has an asymptomatic bartholin duct cyst >> they drain into the vulvar vestibule at the *4 & 8 o'clock positions* to provide lubrication > Cyst is *soft, mobile, nontender* and found at the base of labia majora > Can be *flesh colored* Condylomata acuminata > Results from HPV > Growths would be described as Exophytic or sessile, & either solitary or multiple >> *but not cystic masses*

A 19F, presents d/t continuous vaginal discharge. The pt has had a clear, slightly malodorous vaginal discharge for the past few weeks but no pelvic pain, dysuria, or vulvovaginal pruritus. She is 6 wks postpartum from a vaginal delivery of a stillborn male infant after 3 days of labor & 4hrs of pushing. The pt had no prenatal care & delivered at home. Temp is 98F, BP is 120/60, Pulse is 70/min. PE shows a well-healed, 3rd degree perineal laceration. On speculum, there is a small, red area of granulation tissue on the anterior vaginal wall. The cervix has no lesions & there is a pool of clear vaginal fluid in the vagina w/ a pH of 6. Which of the following would most likely establish the Dx? (Bladder dye test OR Wet mount microscopy)

Bladder dye test > pt has a *Vesicovaginal fistula* >> a complication of obstructed labor >> typically presents w/: 1- Young females in areas of poor prenatal care 2- W/in the first few weeks postpartum 3- *Continuous vaginal discharge w/ alkaline pH* 4- Malodorous discharge d/t surrounding necrotic tissue PE findings 1- *Vaginal pooling of urine* 2- A visible defect *area of raised, red granulation tissue* on the anterior vaginal wall Testing > *Bladder dye test* to confirm Dx TMT > surgery Wet mount microscopy > Used to Dx B.V. or Trichomonas >> *Would not cause vaginal pooling or granulation tissue*

A 39F, G3P2, presents to the ED at 37 wks d/t leakage of fluid for the last 2 hours. She has also had some vaginal bleeding & contractions for the last hr. Fetal mvmts are normal. The pt immigrated to the US 6 months ago & has had no prenatal care this pregnancy. Her previous pregnancies were uncomplicated & delivered vaginally at term. BP is 100/60, Pulse is 102/min, & RR is 16/min. On speculum exam, rupture of membranes is confirmed & the pt has some small blood clots at the cervical os. An US reveals an active fetus in cephalic presentation whose growth is consistent w/ gestational age & placental tissue covering the cervix. Fetal HR baseline is 150/min w/ accelerations & no decels. Which of the following is the most appropriate next step in MGMT? (administer Anti-D immune globulin OR CS delivery)

CS delivery > Pt has placenta previa >> *CS delivery indicated after 36-37 wks* Administer Anti-D immune globulin > Pt requires CS to present hemorrhage >> Anti-D can be given after delivery

A23F, Primigravid, at 38 wks, presents to the ER for spontaneous rupture of the membranes & painful contractions. The pt has had an uncomplicated prenatal course & a negative GBS culture last week. Leopold maneuvers show an estimated fetal wt of 8lbs. Cervical exam shows the cervix to be 8 cm dilated & 90% effaced w/ the head at 0 station. The pt is admitted. Epi is administered, after which the pt reports increasing rectal pressure; cervical exam at that time is unchanged. An intrauterine pressure catheter reveals contractions every 2-3 mins. The sum of the contractions strengths over 10 mins is 220 montevideo units. 4 hrs later, the cervix remains unchanged. Fetal heart tracing is normal. Which of the following is the most appropriate next step in MGMT? (Cervical exam in 2 hours OR CS OR IV oxytocin)

CS delivery > Pt is in active stage >> pts who are *>/= 4-8cm dilated w/ slow dilation or absence of further dilation (labor arrest) have an active phase abnormality* >> this occurs when 1- there is no cervical change for >/= 4 hrs w/ adequate contractions 2- No cervical change for >/= 6 hrs with inadequate contractions *TMT is CS delivery* Oxytocin > It is indicated in protracted labor (cervical change slower than expected +/- inadequate contractions) *this pt has adequate contractions* Cervical exam in 2 hrs > this pt already meets criteria for *active phase arrest of labor* waiting another 2 hrs would be appropriate if her contractions were inadequate

A 32F, presents d/t several days of vaginal itching & discharge. She also has pain w/ intercourse. The pt has had no fever, chills, abd pain, urinary frequency, or dysuria. In the past year she has had 2 new male sexual partners. She takes oral contraceptives and uses condoms on most occasions. The pt has no other chronic med conditions & has had no surgeries. She takes no other daily meds. The pt drinks several glasses of wine each week but does not smoke or use drugs. Her last menstrual period was 3 wks ago. Temp is 98F, BP is 100/60. Pelvic exam shows vulvar & vaginal erythema & a small amount of vaginal discharge w/ a pH of 4. Which of the following is the most likely causal organism? (Candida OR Chlamydia)

Candida > pt presents w/ *vulvovaginal erythema & vaginal discharge* > can be *triggered by estrogen containing contraceptives* > Candida can present w/: 1- *Vuvlar pruritus* (+/- excoriations) 2- Discharge 3- Dyspareunia 4- Dysuria 5- *Normal pH 3.8-4.5* Chlamydia > can cause acute cervicitis >> however typically presents w/: 1- *Mucopurulent cervical discharge* (no vulvovaginal erythema) 2- *cervical friability* 3- +/- intermenstrual or post-coital bleeding 4- Dyspareunia 5- Dysuria 6- Vaginal irritation

A 32F, presents d/t several days of vaginal itching & discharge. She also has pain w/ intercourse. The pt has had no fever, chills, abd pain, urinary frequency, or dysuria. In the past year she has had 2 new male sexual partners. She takes oral contraceptives and uses condoms on most occasions. The pt has no other chronic med conditions & has had no surgeries. She takes no other daily meds. The pt drinks several glasses of wine each week but does not smoke or use drugs. Her last menstrual period was 3 wks ago. Temp is 98F, BP is 100/60. Pelvic exam shows vulvar & vaginal erythema & a small amount of vaginal discharge w/ a pH of 4. Which of the following is the most likely causal organism? (Candida OR Chlamydia)

Candida > pt presents w/ vulvovaginal erythema & vaginal discharge >> can be triggered by estrogen containing contraceptives >> Candida can present w/: 1- Vuvlar pruritus (+/- excoriations) 2- Dyspareunia 3- Dysuria 4- *Normal pH 3.8-4.5) Chlamydia > can cause acute cervicitis >> however pts typically have a *mucopurulent cervical discharge* but no vulvovaginal erythema

A 22F, presents for her first gynecologic exam & a discussion about contraception. Menarche was at age 13. Last menses was 2 wks ago; periods occur every 30 days & last for 4 days. She has no dysuria, urinary frequency, vaginal discharge postcoital spotting, or abd pain. She became sexually active a few months ago w/ her bf; she is monogamous & currently uses condoms for contraception. The pt has PSHx of a yeast infection at age 18 after taking antibiotics for a throat infection. She currently takes no meds & has no allergies. She does not smoke, drink, or use drugs. The pt has received all vaccinations except the HPV series. Her mother, Dx'd w/ ovarian cancer at age 46, is currently undergoing chemo. Vitals are normal. Complete PE, including pelvic exam, is normal. In addition to her first Pap, which of the following is the best recommendation for this pt? (Cervical HPV testing OR Cervical swab for chlamydia & gonorrhea)

Cervical swab for chlamydia & gonorrhea > Pt is sexually active, and these are the two most common bacterial STD's > high prevalence in women < 25 yrs old > Often have no symptoms *Sexually active women < 25 are advised to screen annually* Cervical HPV testing > *Not recommended for ages 21-29* as most HPV infections spontaneously clear in pts >> instead should be offered HPV vaccinations

A 26F, presents d/t concerns about worsening symptoms of PCOS. The pt was Dx'd w/ PCOS 6 yrs ago after eval of abnormal uterine bleeding & lip & chin hair. She was prescribed daily combined estrogen/progestin oral contraceptives, which regulated her menses. However, for the last 2 months, she has had irregular menstrual bleeding despite taking her pills as directed. The pt has had no changes in hair growth, wt gain, or galactorrhea but has noticed some changes in her voice. She has no other chronic medical conditions & takes no other meds. BP is 132/84, Pulse is 78/min, BMI is 33. Velvety, pigmented plaques are noted on the back of the neck and the axilla. A few coarse hairs are noted on the upper lip & chin. Multiple small comedones are on the forehead but not on the chest of upper back. The abd is soft and nontender w/ no palpable masses or terminal hair. An US is ordered d/t suspicion of an ovarian tumor. Which of the following is most concerning for an ovarian tumor in this pt? (Changes in voice OR Menstrual bleeding cycle)

Changes in voice > Most likely cause of the pt's symptoms is an androgen secreting tumor (new onset voice deepening) >> Most likely d/t sertoli-Leydig cell tumor that secretes high testosterone levels Menstrual bleeding cycle > can be seen w/ androgen secreting tumors but not specific

A 32F, Presents d/t difficulty conceiving despite frequent, unprotected sex w/ her husband over the last 3 years. She had multiple sex partners in the past & never became pregnant despite inconsistent condom use. Menses started at age 12 & are irregular. The pt has had recurrent vaginal candidiasis for which she uses an OTC vaginal suppository. Vitals are normal. The thyroid is not enlarged. She has thick, dark, velvety plaques under her axillae. Brest exam is normal. Abd exam shows no masses. Pelvic exam shows normal ext genitalia; a small, mobile uterus w/ no cervical motion, tenderness, & bilaterally enlarged ovaries. TSH, LH, & prolactin levels are normal. Which of the following is the most appropriate therapy for this pts infertility? (Clomiphene citrate OR Cyclic progesterone)

Clomiphene citrate > Pt presents w/ irregular menstruation, infertility, & enlarged ovaries >> also has acanthosis nigricans which indicated insulin resistance >> all point to *PCOS* > PCOS infertility TMT is 1- *clomiphene* for ovulation induction 2- Oral contraceptives >> to treat menstrual irregularities 3- Weight loss *First line* Cyclic progesterone > May be given for endometrial protection form uncontrolled anovulatory proliferation of the lining in pts w/ PCOS >> *does not facilitate ovulation*

A 26F, nulliparous, presents d/t severe pain during sex. The pain is particularly unbearable w/ deep penetration. She is in a monogamous relationship w/ her husband of 2 years but has been avoiding sexual intimacy d/t pain. The pt's husband is her first sexual partner, & these symptoms have caused significant strain in their marriage. Menses are painful, occur every 30 days, and last 5 days. The pt also has pain when passing stool as well as sporadic pelvic pain that waxes and wanes w/ no discernible trigger. She and her husband are not planning on having children for another few years. TVUS shows no abnormalities. Which of the following is the most appropriate TMT for this pt's condition? (Oral contraceptives OR Oil based lubes)

Combo oral contraceptives > pt presents w/ signs of *endometriosis* >> hallmark signs are: 1- *Dysmenorrhea* (Sig pain w/menses) 2- *Dyspareunia* (pain on deep penetration d/t implants on the posterior cut-de-sac 3- *Dyschezia* (pain w/ defecation) TMT *Combo oral contraceptives* Oil based lubes > Only if pt was older and/or had signs of menopause

A 34F, presents for eval of vulvar lesions that have worsened over the past 3 months. She has had 2 sexual partners in the last year. Medical Hx includes a genital infection w/ HSV Dx'd at age 23; when she was 25, her sexual partner was diagnosed w/ syphilis. Cervical cancer screening last year revealed atypical squamous cells of undetermined significance, w/ negative testing for high-risk HPV. The pt smokes but does not use alcohol or drugs. Exam reveals multiple nontender, fleshy, verrucous growths clustered at the vestibule of the vulva & over the labia majora. A few lesions are friable & bleed on manipulation. The remainder of the exam is normal. Which of the following is the most likely Dx? (Condylomata acuminata OR Genital herpes OR Lichen Planus

Condylomata acuminata > HPV 6 & 11 causes *anogenital warts* that appear as *Soft fleshy (skin colored) lesions in the internal or external vaginal, vuvlar, & anal regions* - Generally are asymptomatic & nontender, although *pruritic, friable lesions (that may bleed w/ manipulation)* may occur - RF include > *Chronic tobacco use* > Immunosuppression Genital herpes >Appear as tender ulcerative blisters or pustules *(not fleshy, verrucous growths)* Lichen Planus (6p's) > Presents a *Planar, polygonal, Purple, pruritic, papules & plaques* w/ white labial lesions overlying white reticular lines (Wickham striae)

A 38F, G1A1, presents for a follow-up a week after suction curettage. The pt initially presented 7 wks after her last menstrual period for an initial prenatal appointment. At that time, she had breast tenderness but no abd pain, nausea, or vaginal bleeding. During that visit, US revealed an empty gestational sac. B-hCG level was 53,256. The pt returned a wk later & repeat US confirmed a missed abortion. Suction curettage was performed &. the pathological specimen from the procedure indicated a Dx of hydatidiform mole. Today, the pt has minimal bleeding but no pain, fevers, chills, or abnormal vaginal discharge. BP is 110/80, and wt is 121.3 lbs. Pelvic exam shows minimal dark red blood in the vaginal vault & no active bleeding. On bimanual exam, the uterus is small & nontender. No adnexal Masses are palpated. B-hCG level is 1412. Which of the following is the best next step in mgmt? (Contraception for at least 6 months OR Serial CA-125 levels)

Contraception for at least 6 months > MGMT for a hydatidiform mole involves: 1- Uterine evac via suction curettage 2- Surveillance for the development of GTN >> During this period *contraception is prescribed* as pregnancy would make it hard to determine the significance of a rising B-hCG level and to treat GTN >> GTN rarely develops 6 months after a suction curettage so only need it fo 6 months Serial CA-125 levels > Not used for GTN >> used only for ovarian cancer

An 18F, nulligravid, Presents for emergency contraception. She does not want to become pregnant after a condom broke during sexual intercourse the previous night. The pt and her longtime bf have always used condoms, & their evaluations for STD's have been negative. Her last menstrual period was 2 wks ago; she has irregular menses. The pt takes no meds & does not drink, smoke, or use drugs. Vitals and PE are normal. A pregnancy test is negative. Which of the following is the most effective emergency contraceptive method for this pt? (Copper IUD OR Levonorgestrel pill OR Ethinyl estradiol + levonorgestrel pills)

Copper IUD > 99% effective >> impairs implantation via inflammatory rxn >> may be inserted up to 5 days following unprotected sex and age and parity are now contraindications Levonorgestrel pill > 85% effective >> Designed for emergency contraception only>> but are less effective than Copper IUD Ethinyl estradiol + levonorgestrel > 75% effective Combo therapy >> may be used for emergency contraception >> multiple pills must be taken simultaneously to achieve a progestin does that delays ovulation >> but least effective of the three methods

A 32F, G5P4, presents for a follow-up of breast biopsy that confirmed high-grade malignant features. 9wks ago, the pt was incidentally found to have a Brest mass during a pregnancy visit. The pregnancy ended in miscarriage. She is coping w/ the new Dx & others has been feeling well w/ no vaginal bleeding of abd pain. FHx is sig for ovarian cancer in her mother, who died at age 50. Genetic testing reveals a mutation in BRCA2 gene. The pt is scheduled to begin chemo & is advised to avoid pregnancy. She took oral contraceptives during adolescence w/ no side effects. Breast exam shows a well-healing biopsy scar. Which of the following is the best contraceptive for this pt? (Copper IUD OR Progesterone IUD)

Copper IUD >> Most effective & not contraindicated in the setting of Breast cancer Progesterone IUD > CI in the setting of breast cancer b/c of potential absorption of progesterone >> *All Hormone containing contraception is contraindicated in pt's with breast cancer*

A 21F, Presents to the ED d/t to severe right lower quadrant pain. The pain began 3 hours ago & was intermittent & dull. However, over the past 4 hrs the pain has become constant & severe. She is now having nausea & vomiting, but no diarrhea or dysuria. The pt has no chronic med conditions & has had no surgeries. She has regular monthly periods, & her last period was 2 wks ago. Temp is 100F, BP is 140/70, Pulse is 93/min. BMI is 19/. PE shows scant physiologic cervical discharge & no cervical motion tenderness. A tender, right adnexal mass is palpated. Pregnancy test is neg. Pelvic US reveals a 6-cm, partially calcified, right ovarian mass w/ multiple thin, echogenic bands. There is decreased Doppler flow to the right ovary. Which of the following is the most likely cause of this pt's mass? (Cystic teratoma OR Tubo- ovarian abscess OR Follicular cyst)

Cystic teratoma > Mature cystic teratomas *AKA dermoid cysts* are common in premenopausal women US findings > partially calcified mass (teeth) w/ multiple thin, echogenic bands (hair) Tubo-ovarian abscess > Tend to present w/ 1- Fever (pt has low-grade fever) 2- Tender, multiloculated, cystic mass w/ distortion of normal adnexal structures 3- Abnormal discharge 4- Cervical motion tenderness Follicular cyst > Usually asymptomatic >> may have pelvic pain mid-cycle due to rupture 1- Unilateral pelvic pain mid-cycle (Mittelschmerz) 2- Simple, small, thin-walled cyst +/- free fluid (Physiologic)

A 42F, G4P4, presents for her routine health maintenance exam. The pts' menses occur every 29 days w/ 2 days of heavy flow followed by 3 days of light flow. Her last menstrual period was a week ago & she has not experienced any recent changes to her menstrual cycles. Last month, she had a normal screening mammogram. The pt had 4 children via a normal vaginal delivery & breastfed each child until age 1. Her mom had breast cancer at age 67 & is doing well after mastectomy & chemo. The pt is an aerobics instructor & teaches classes 3 times a week. She does not smoke & drinks 2 glasses of wine w/ dinner every evening. She is 5ft 5in tall & weighs 143 lbs, BMI is 23. Breast exam shows no masses or discharge. She is concerned about her likelihood of getting breast cancer. Which of the following is the best recommendation for this pt? (Decrease alcohol intake OR Genetic mutation testing OR Initiate oral contraceptives)

Decrease alcohol intake > Modifiable RF for breast cancer include: 1- Hormone replacement therapy 3- Nulliparity 4- Increased age at first live birth 5- Alcohol consumption Initiate oral contraceptives > Decreases the risk of *ovarian cancer* not breast cancer Genetic mutation testing > Can first be performed in affected individual w/ characteristics of a carrier *breast cancer Dx at age <50, ovarian cancer at any age*

A 42F, G1P1, presents w/ abnormal uterine bleeding. The pt underwent menarche at age 15 & had irregular menses that normalized w/ oral contraceptives until 4 years ago, when she became pregnant & had a CS w/ tubal ligation. The pt's menstrual cycles after delivery occurred monthly w/ 4-5 days of moderate bleeding. However, over the last year, her menses have become increasingly irregular & now occur every 2-3 months, w/ heavier bleeding & passage of clots. She also has difficulty concentrating at work d/t increasing irritability & depressed mood. She has no headaches or hot flashes. Her mom & older sister underwent menopause at age 49. Vitals are normal. BMI is 26. Skin is cool & dry. PE reveals a well-rugated vagina, mobile uterus, and small, nontender ovaries. The remainder of the PE is normal. Pregnancy test is neg. The pt's labs would most likely reveal which of the following? (FSH, TSH, Prolactin - all normal OR Decreased FSH, Increased TSH, Increased prolactin

Decreased FSH, Increased TSH, & increased Prolactin > Pt has *hypothyroidism* >> pts present w/: 1- *Cool, dry skin* 2- Difficulty concentrating 3- Depressed or increased irritability Physiology >> Increased TRH anterior pituitary to release TSH & Prolactin >> Hyperprolactinemia inhibits the Hypothalamus & anterior pituitary causing decreased FSH & LH levels >> resulting in anovulation & abnormal uterine bleeding Normal FSH, TSH, Prolactin > seen in PCOS

A 37F, Presents for eval of infertility. She & her 39 ys husband have not been able to conceive after 18 months of unprotected & frequent intercourse. Menstrual cycles occur every 28 days, last 5 days, & have heavy bleeding & cramping the first day. The pt had a miscarriage at age 27 that required a dilation & curettage. The couple had another spontaneous pregnancy 6 ys later that resulted in an uncomplicated term vaginal delivery. The pt feels well & has no medical conditions. She has never had an STD. She doesn't smoke, drink, or use drugs. The pt is an aerobics instructor & teaches two 60 min classes daily. BP is 120/80, Pulse is 84/min. BMI is 23. She has no thyromegaly. Breast exam shows no palpable masses, axillary lymphadenopathy, or expressed nipple discharge. Pelvic exam reveals a normal ext genitalia, a well-rugated vagina, a mobile uterus & normal ovaries. Which of the following is the most likely underlying cause of the infertility? (Decreased ovarian reserve OR Uterine synechiae)

Decreased ovarian reserve > In women w/ regular menstrual cycles & older than 35, infertility can occur d/t *diminished ovarian reserve* characterized by decreased oocyte number & quality Uterine Synechiae > Can occur after dilation & curettage >>but pt was pregnant and delivered a baby after that so this is unlikely

A 29F, presents to the ED w/ fever, chills, & lower abd pain. Two days ago, the pt underwent dilation & curettage for a 9-wk missed abortion. After the procedure, she initially had light spotting, but the bleeding has become increasingly heavy & malodorous w/in the last few hs. She has developed increasing abd pain & fever unrelieved by acetaminophen. The pt has no chronic medical conditions & has had no other surgeries. Temp is 103F, BP 88/50, Pulse is 118/min. On speculum exam, the cervix is visibly 1cm dilated & has purulent discharge at the os. The uterus is enlarged & has cervical motion tenderness. Transvaginal US shows a thickened endometrial stripe. Urine pregnancy test positive. The pt is started on bread spectrum antibiotics & undergoes suction dilation & sharp curettage. The pt is at increased risk for which of the following conditions? (Development of intrauterine synechiae OR Dysplastic changes to the cervical epithelium)

Development of intrauterine synechiae > Clinical features - Abnormal uterine bleeding - Amenorrhea - Infertility - Cyclic pelvic pain - Recurrent pregnancy loss RF - Infection - Intrauterine surgery (*curettage*, myomectomy) Dysplastic changes in the cervical epithelium > Associated w/ HPV >> *no association w/ intrauterine surgery or infection*

A 32F, at 14 wks presents for an initial prenatal visit. She has had no headaches, Chest pain, abdominal pain, or LE swelling. Medical Hx is sig for DM I managed w/ multiple daily insulin injections. BP is 154/94, Pulse is 86/min. Fetal heart tones are 152/min on Doppler. Urinalysis shows 2+ protein but no red or white blood cells. Which of the following is the most likely cause of this patients proteinuria? (Diabetic nephropathy OR Gestational HTN OR Preeclampsia)

Diabetic nephropathy > PT has a Hx of DM I & presents w/ proteinuria & Elevated creatinine 1.4 >> *The development of significant proteinuria (> 300mg/day; 1+ protein) prior to 20 wks suggests underlying renal disease* Gestational HTN > Causes new onset HTN *exclusively at >20 wks* >> *no proteinuria* Preeclampsia > Causes new onset HTN *& proteinuria* >> *Exclusively at >/=20 wks*

A 28F, G2P1, at 38 wks is admitted to L & D d/t prelabor rupture of membranes. Clear amniotic fluid has been leaking continuously for the last 12 hs. The pt has irregular, non-painful contractions & no vaginal bleeding. Prenatal care during this pregnancy has been uncomplicated, & the pt's previous pregnancy ended in a term vaginal delivery. Temp is 98.1F, BP is 120/70, Pulse is 78/min. The cervix is dilated 3cm & 50% effaced w/ the fetal vertex at -2 station. Fetal heart tracing on admission is as seen (normal with moderate variability and Accelerations, no decels). After an oxytocin infusion is started for labor augmentation, the pt begins to have regular, painful contractions that require epidural anesthesia. Two hs later, temp is 99F, BP is 120/80, pulse is 80/min. On repeat exam the cervix is now 7 cm dilated, 100% effaced, w/ fetal vertex at +1 station; and current fetal heart tracing shows Late decels. Which of the following is the best next step in MGMT? (Discontinue uterotonic agents OR Initiate amnioinfusion)

Discontinue uterotonic agents > Uterine contractions temporarily interrupt intervillous blood flow >> and excessive contractions (d/t oxytocin) may cause fetal compromise as a result of decreased uteroplacental blood flow during contraction & inadequate recovery time Initiate amnioinfusion > Would be correct is *variable decels* (caused by Cord compression) were visible

A pt has been Dx'd w/ syphilis infection but her serum RPR & HSV PCR tests are both negative. Which of the following is the best next step in MGMT? (Administer Empiric ceftriaxone & Az OR Empiric penicillin)

Empiric penicillin > Pt has Syphilis infection >> *which may have neg RPR d/t early infection* Administer Ceftriaxone and Az > Used to treat Nesseria Gonorrhea

A 56F, presents for a routine health maintenance exam. The pt is feeling well & has no concerns. She has been postmenopausal for 4 years & is not using hormone therapy. The pt had a normal screening mammogram last year. A Pap test 3 years ago was normal, but HPV testing was not performed. The pt has well-controlled HTN and hyperlipidemia. She has had 2 CS deliveries & a tubal ligation w/ her last delivery at age 30. BP is 130/90 & pulse is 72/min. BMI is 38. PE is normal. Speculum exam shows no cervical abnormalities. A Pap test shows Benign endometrial cells & no intraepithelial lesions. Which of the following is the best next step in MGMT? (Endometrial biopsy OR No test needed)

Endometrial biopsy > Pt is postmenopausal + Pap test that shows *benign appearing endometrial cells* > pt is > 45 > Pt is obese > postmenopausal > benign appearing endometrial cells should not be seen in pap

A 42F, nulliparous, presents d/t abnormal uterine bleeding. The pt previously had regular monthly menses but, for the last 8 months, has had irregular bleeding & spotting that last 1-5 days. She initially attributed this irregular bleeding to stress but is now having abd bloating & daily breast tenderness. The pt has no chronic med conditions & Pap test 2 yrs ago was normal. She does not smoke, drink, or use drugs. The pt recently married & is hoping to conceive w/in the next several months. Vitals are normal. BMI is 30. Brest exam shows bilateral diffuse tenderness & no masses. There is fullness in the LLQ w/ no rebound or guarding. PE shows a large, non-tender left adnexal mass. A urine pregnancy test is negative, Pelvic US reveals a 10-cm, complex left ovarian mass & an irregular endometrial stripe. Which of the following is the best next step in MGMT? (Endometrial biopsy OR Progestin IUD)

Endometrial biopsy > Pt presents w/ signs of *granulosa cell tumor* >> which presents w/: 1- *Breast tenderness* 2- *Abd bloating/pain* (mass effect, ascites) 3- *Abnormal uterine bleeding* (b/c of uncontrolled endometrial proliferation b/c of estrogen) >> want to do an *endometrial biopsy* d/t the possibility of concomitant endometrial hyperplasia/cancer from excessive estrogen) Progestin IUD > is a TMT option for abnormal uterine bleeding >> but pt is at *high risk for endometrial cancer* so she must have biopsy

A 38F, nulligravid, presents d/t a menstrual period that has lasted longer than usual. Her menstrual period began 11 days ago & the bleeding has been intermittently heavy, requiring 3 pad changes daily. The pt's prior menstrual period was 3 months ago. She stopped taking oral contraceptives 5 ys ago & since then her menstrual periods have become irregular & unpredictable. She has had no headaches or visual changes. The pt has DM II treated w/ oral meds. She has no Hx of abnormal pap, her most recent test was a year ago. She does not use tobacco, alcohol, or drugs. Temp is 99F, BP 126/76. Pelvic exam is limited by the pt's obesity but shows dark red blood in the posterior vaginal vault; no active bleeding is seen. Labs show: - Hb ------------10.2 - Platelets ------240K - Prolactin ------5ng/mL - TSH -----------1.8 uU/mL Urine pregnancy test is neg. Pelvic US reveals a small uterus w/ a 3mm endometrial stripe, no fibroids, and no adnexal masses. Which of the following is the next best step in MGMT? (Cyclic progestins OR Endometrial biopsy OR Endometrial ablation OR Combined oral contraceptives)

Endometrial biopsy > indications for endometrial biopsy - *Abnormal uterine bleeding (bleeding that lasts > 7 days)* >> AUB is the most common presenting clinical feature of *endometrial hyperplasia & cancer* - Endometrial biopsy is performed in *pts <45 who have AUB* & are at risk for endometrial hyperplasia & cancer from unopposed estrogen d/t 1- *obesity* and/or 2- *chronic anovulation* (PCOS) Endometrial ablation > surgical procedure that destroys the endometrium >> it is the TMT option for irregular, heavy bleeding in premenopausal women that can be performed only after a benign (negative) biopsy Cyclic progestins & combined OCP's > In premenopausal pts, first-line options for MGMT of AUB include either combined estrogen/progestin or cyclic/continuous progestins >> However these therapies are initiated only after endometrial cancer & hyperplasia have been excluded via biopsy

A 27F, Nulliparous presents for eval of intermittent left pelvic pain over the last 8 months. She has noticed that exercise exacerbates the discomfort. The pt is sexually active w/ her husband & stopped taking OCP's two years ago w/ the intention of having kids. Last menstrual period was 2 wks ago. The pt's cycles are around 27 wks w/ bleeding that lasts 4 days. She had trichomoniasis when she was an adolescent. Temp is 98.9F, BP is 120/72. PE shows a normal sized uterus & an enlarged left adnexa. US reveals a homogenous cystic appearing mass on the left ovary but is otherwise normal. Which of the following is the most likely Dx? (Endometriosis OR Epithelial ovarian cancer)

Endometriosis > Endometriosis can cause chronic pelvic pain (>6 months) in reproductive aged women >> Pain symptoms are unique to women but most present w/: 1- Dysmenorrhea 2- *Non-cyclic pain that can be exacerbated by exercise* 3- Infertility PE findings 1- Fixed & immobile uterus 2- Rectovaginal nodularity US findings - Adnexal mass >> *Homogenous cystic ovarian mass is suggestive of endometrioma* >> which can be the ONLY clinical manifestation is some women Epithelial ovarian cancer > Unlikely in young woman > Malignancy would show >> *Septated mass w/ solid components* on US

A 60F, G3P3 presents d/t SOB. Over the past 6 months, the pt has been finding it progressively more difficult to take deep breaths. She has also noticed that she cannot fit into any of her pants lately despite a decreased appetite & nausea. The pt has DM II and no major surgeries. All her children were born vaginally. Routine Mammography 4 yrs ago was normal. The pt has a sister w/ breast cancer, BRCA mutation positive. PE shows an ill-appearing woman w/ a distended abdomen & decreased bowel sounds. The lungs are clear to auscultation. PE shows a firm, nodular, non-mobile mass in the left adnexa. A pelvic US is performed & confirms the PE finings. Which of the following is the most appropriate next step in MGTM? (Exploratory laparotomy OR Image-guided biopsy of pelvic mass)

Exploratory laparotomy > Pt most likely has *epithelial ovarian carcinoma* >> which occurs in postmenopausal women w/ a FHx of breast cancer (BRCA positive) >> Usually presents w/: 1- Early *nonspecific symptoms Bloating, abd pain* 2- Ascites *(SOB, Decreased appetite, abd distention, decreased bowel sounds)* >> PE findings - *Firm pelvic mass w/ nodularity* TMT is *exploratory laparoscopy* w/ cancer resection & inspection for the entire abd cavity for mets is required* Image-guided biopsy of pelvic mass > CI in pt b/c of malignant features seen in pt, including (FHx, Ascites, dyspnea, mass w/ firm nodularity) >> biopsy can lead to rupture of the mass & spreading

A 28F, G0P0, presents w/ her husband, age 38, for evaluation of infertility. The couple has not conceived despite frequent intercourse w/out contraception for over a year. Menarche was at age 18. The pt's menstrual cycles are irregular, & her last period was 8 wks ago. She had no nipple discharge. She had trichomoniasis as a teenager. STD testing at her most recent exam was neg. Her husband had a normal semen analysis. The pt takes a prenatal vitamin and does not smoke, drink, or use drugs. BMI is 28. BP is 130/80 & pulse is 84/min. Heart & lung exam is normal. The abd is soft and non-tender w/ no palpable masses. Pelvic Exam shows a normal ext genitalia & a mobile uterus w/out adnexal masses. Urine pregnancy is neg. Serum TSH & prolactin levels are normal. Free testosterone is elevated. Which of the following is the most likely cause of this couple's infertility? Explain the physiology. (Primary ovarian insufficiency OR Failure of follicle maturation)

Failure of follicle maturation > pt has signs of PCOS - Infertility - irregular menses, - *Hyperandrogenism* >> shown as elevated testosterone Pts have an imbalance of LH/FSH release (LH > FSH) but results in lack of LH surge >> leading to *failure of follicle maturation* Primary ovarian insufficiency > Pts may present w/ infertility & oligomenorrhea >> they usually also have a concomitant autoimmune disorder and *normal testosterone levels*

A 26F, G2P1m at 35 wks, presents to the L & D d/t regular, painful contractions over the past 3 hrs. The pt has had no prenatal care during this pregnancy. She has no chronic medical conditions, & her only surgery was a low transverse CS delivery 2 yrs ago. Her cervix on admission is 7 cm dilated & 100% effaced w/ fetal head at 2+ station. Fetal HR tracing on admission is category 1. Epidural is administered, & the pt has pain relief from the contractions. ROM results in bright-red amniotic fluid. BP is 130/80, & pulse is 112/min. Current fetal monitoring shows - Fetal heart rate at 135/min w/ no accelerations and no decls. - Sinusoidal fetal heart tracing [smooth, wave-like oscillation (rolling line) w/ fixed amplitude (5-15) & frequency of 3-5 cycles/min Which of the following is the most likely cause of this fetal heart rate tracing? (Fetal blood loss OR Preterm gestation OR Oligohydraminos)

Fetal Blood loss > *Sinusoidal tracings* are associated w/ *severe fetal anemia* >> suggested in this pt w/ likely *fetal blood loss from ruptured vasa previa* Preterm gestation > May be associated w/ *Increased FHR baseline, reduced variability, & lower amplitude accelerations* >> But does not cause sinusoidal rhythms Oligohydraminos > May cause *variable decels* d/t umbilical cord compression

The pt undergoes induction of labor & vaginal delivery of a stillborn male fetus w/ no gross abnormalities. Review of the pt's prenatal records shows that the pt missed several appts & was last evaluated at 20 wks. The pt asks how to prevent this from happening in the future. In addition to comforting the pt & offering emotional support, which of the following is the most appropriate response? (Consistent prenatal care could have prevented this outcome OR Fetal autopsy & placental eval may provide more info)

Fetal autopsy & placental eval may provide more info > Thorough maternal, fetal, & placental evals are necessary to determine the etiology of intrauterine fetal demise Consistent prenatal care could have prevented this outcome > Cannot concludes that consistent prenatal care would have prevented the outcome & it's not a sympathetic answer choice

A 36F, G1P0, at 26 wks presents for a routine prenatal office visit. The pt reports good fetal mvmt & has no concerns. She initiated prenatal care at 12 wks gestation & an US at that time was consistent w. her last menstrual period. She declined all genetic testing. Anatomy US performed at 19 weeks revealed an estimated fetal wt consistent w/ 18 wks gestation & an atrial septal defect. The pt has a Hx of depression for which she takes citalopram. She quit smoking prior to conception & does not drink or use drugs. Pregnancy BMI was 23 & her wt gain has been appropriate. BP is 138/89. Fundal ht is 22cm & fetal heart tones are heard by doppler. An US reveals fetal head circumference, abd circumference, and estimated fetal wt at the 4th percentile. Normal amniotic fluid level & a fundal placenta are present. A biophysical profile is 8/8. Umbilical dopplers are normal. Which of the following is the most likely cause of these fetal findings? (Fetal chromosomal abnormality OR Fetal congenital infection)

Fetal chromosomal abnormality > Baby is showing Fetal Growth Restriction (FGR) shown by the fetal wt only being in the 10th percentile >> Causes of FGR are 1- *fetal chromosomal abnormalities* 2- First trimester/congenital infections >> *Atrial septal defect + pts age* are more likely to be relate to Trisomy 21 (downs) >> *Downs associated w/ increased maternal age* Fetal congenital infection > Would typically follow maternal illness (moms not sick) > also would typically reveal multiple abnormalities (intracerebral calcifications - toxo, Ventriculomegaly- CMV)

A 33F, G2P1, at 40 wks, is admitted d/t contractions & spontaneous rupture of membranes. The pt underwent a CS w/ her first child d/t breech; this pregnancy has been uncomplicated. She has no medical problems & is taking only a prenatal vitamin. Her pre-pregnancy BMI was 17 & she gained 35 lbs during pregnancy. BP is 130/80. The pt is admitted, epi is administered, & she quickly dilates to 10 cm w/ the fetal vertex at 0 station, occiput transverse. Contractions occur every 2-3 mins & are firm on palpation, & the pt pushes w/ each one. 3 hours later, the PE is unchanged but there is molding of the fetal head. Which of the following is the most likely etiology for this pt's clinical presentation? (Fetal malposition OR Fetal malpresentation)

Fetal malposition > Pt presents w/ *arrest in second stage of labor* >> criteria include 1- no fetal descent after pushing for >/= 3 hrs in nulliparous pts 2- No fetal descent after pushing for >/= 2 hours in multiparous >> *most common cause is fetal malposition* Fetal malpresentation > refers to non-vertex presentation (face, breech) *This pt as vertex position, which is normal*

A 34F, G3P2, at 33 wks presents to ED d/t continued leakage of fluid. The pt reports intermittent vaginal spotting but has had no contractions. Fetal mvmt is normal. Her prenatal course has been complicated by GBS treated in the 1st trimester. The pt has no other chronic medical conditions or previous surgeries. Temp is 102.5F, BP is 90/56, Pulse is 109/min. Fundal Ht measures 30cm. Speculum exam shows pooled fluid in the posterior fornix that turns nitrazine paper blue; the cervix appears visibly closed. A ferning pattern is visualized on microscopy. Fetal HR monitoring shows a rate of 170/min w/ moderate variability. Tocometry shows no contractions. A Transabdominal US reveals a fetus in transverse lie; the single deepest vertical pocket of amniotic fluid is 1 cm. Which of the following is an indication for delivery in this pt? (Fetal presentation OR Fetal tachycardia)

Fetal tachy > Pt has PPROM + Chorioamnionitis >> which presents w/ 1- Fever for mom AND 2- Tachy in baby 3- purulent amniotic fluid 4- Maternal leukocytosis TMT > If pt has *complicated PPROM* (hypotension, Tachycardia) >> *Delivery is indicated* > If pt has PPROM w/out complications >> 1- Corticosteroids 2- Prophylactic Broad spectrum (latency) antibiotics Fetal presentation > Pt is only 33 wks >> fetus will probably change position before delivery

The pt starts the recommend therapy & continues routine follow-up. Her 2 hr post-prandial BG levels remain > 140mg/dL despite increasing therapy. At 37 wks, US shows an estimated fetal wt of 8.8 lbs. At 38 weeks, the pt comes to the hospital in spontaneous labor & has a protracted labor course, pushing for 3 hrs. On delivery of the fetal head, the anterior shoulder cannot be delivered w/ standard maneuvers. The pt is instructed not to push, & the head of the bed is lowered. Which of the following is the best next maneuver to aid delivery? (Flex the hips against the abdomen OR Perform midline episiotomy)

Flex the hips against the abdomen > MGMT of shoulder dystocia is *BE CALM* *B* - Breathe; do not push *E* - elevate legs & flex hips against abdomen *McRoberts* *C* - Call for help *A* - Apply suprapubic pressure *L* - enLarge vaginal opening w/ episiotomy *M* - Maneuvers > Deliver posterior arm > Rotate posterior shoulder *Wood screw* - apply pressure to anterior aspect of the posterior shoulder) > Adduct posterior fetal shoulder *Rubin* - apply pressure to the posterior aspect of the posterior shoulder > Mother on hands & knees *Gaskin* > Replace fetal head into pelvis for CS *Zavanelli* Perform midlinie episiotomy > Does not increase the diameter of the bony pelvis & therefore does not relieve impacted shoulder > also, would increase risk of maternal perineal trauma

A 20F, presents for a refill of oral contraceptives. The pt is in a new monogamous relationship that began 6 month ago. The couple also uses condoms as back-up contraception. The pt became sexually active at age 14 & has had 5 diff lifetime partners. Her last menstrual period was 2 wks ago. The pt has regular menses lasting 4-5 days every 28 days. The periods were previously accompanied by menstrual cramps for the first 2 days, as well as the day before. This pain has improved since she started taking oral contraceptives along w/ ibuprofen. The pt did not receive the HPV vaccine and has never had a Pap test or pelvic exam. PMHx in unremarkable. She smokes socially but does not use alcohol of drugs. Vitals are normal. PE shows external genitalia w/out any lesions. What is the most appropriate next step in MGMT of this pt? (Give HPV vaccine OR Perform Pap w/ HPV test)

Give HPV vaccine > Routine vaccination begins at 11-12 and catch up vaccination until age 26 Perform Pap w/ HPV test > Pap test begins at 21

A 60F, presents for evaluation of postmenopausal bleeding. The pt went through menopause at age 53 & did not use menopausal hormone therapy. Four months ago, she noticed some postcoital bleeding that resolved w/in a day. For the last 3 months, the pt has had daily vaginal spotting & sometimes has bleeding similar to the first day of her menstrual cycle. She has obstructive sleep apnea & hyperlipidemia. Pap test last year was normal. The pt does not smoke, drink, or do drugs. BP is 130/80, Pulse is 68/min. BMI is 34. The abd is soft & nontender. Speculum exam shows a small amount of blood in the vaginal vault & a normal cervix. Bimanual exam reveals right adnexal fullness & a slightly enlarged uterus. Hb is 11.8g/dL. Pelvic US shows an 11-cm solid ovarian mass & no free fluid in the pelvis. Endometrial biopsy shows complex hyperplasia w/out atypia. Which of the following is the most likely Dx? (Granulosa cell tumor OR Mature teratoma)

Granulosa cell tumor >Women w/ an adult subtype of granulosa cell tumor have *chronic, unopposed estrogen exposure* >> results in endometrial hyperplasia (or cancer) >> results in *postmenopausal bleeding & abnormal uterine bleeding* Mature teratoma > most common in females 10-30 > Would have tissue from 3 layers (teeth, hair, sebum)

A 7F, presents d/t breast development that has occurred over the past 6 months. Her parents are concerned that she appears older than her classmates. An episode of vaginal spotting 2 months ago resolved after 3 days. She has had no headaches or visual changes. The pt has no chronic med conditions & has had no surgeries. She takes no daily meds & has no allergies. Ht & Wt are at the 98th percentile & 70th percentile, respectively. PE shows tanner stage 3 breast development. The abd is soft & moderately distended & a palpable fullness is present in the pelvic region. Bone age is advanced at age 10. Pelvic US reveals a large right-sided adnexal mass. Which of the following is the most likely Dx? (Granulosa cell tumor OR Sertoli-Leydig cell tumor)

Granulosa cell tumor > Granulosa cells convert testosterone to estrogen and secrete inhibin >> resulting in high estradiol & inhibin levels >> this would result in *precocious puberty* in juveniles > in adult women >> would be associated w/ increased estrogen symptoms/complications Sertoli-Leydig cell tumor > would secrete testosterone >> pt would have virilization

A 24F, presents for her first prenatal visit. Her last menstrual period was 2 months ago. She has a Hx of IV heroin use but stopped when she was Dx'd w/ HIV last year. The pt has completed a methadone program & currently does not smoke, drink, or use drugs. Vitals & PE are normal. Serum labs show: - CD4. 600cells/uL - HIV-1 viral load 2,680 Copies/mL Urine b-hCG is positive & tox screen is neg. US confirms an intrauterine pregnancy. Which of the following is the best method of reducing maternal-fetal transmission of HIV infection? (Nevirapine therapy for infant after birth OR Triple HARRT therapy for mom throughout pregnancy)

HARRT therapy for mom throughout pregnancy Nevirapine therapy for baby after birth > Generally its Zidovudine >> Nevirapine is used is mom did not have HARRT therapy

A 24F, presents d/t vulvar pain & dysuria for the past 2 days. The pt has not had these symptoms before & has had no increased urinary frequency or flank pain. She returned home form a trip to the Caribbean 2 wks ago & had an intermittent fever & malaise for the last week. The pt is sexually active & has had 3 new male partners over the last 6 months. She has no chronic medical conditions, & her only med is a combo oral contraceptive. Vitals are normal. PE shows grouped, shallow ulcers w/ Surrounding erythema on the left labia minora. The left inguinal LN's are tender to palpation. Urinalysis shows 5-10 leukocytes & 1-2 RBC/hpf. Urine culture is negative. Which of the following pathogens is the most like of this pt's symptoms? (Chlamydia OR H.ducreyi OR HSV OR Syphilis )

HSV > Will present as: 1- *Painful* shallow ulcers 2- *Tender lymphadenopathy* H.ducreyi > Single large, *painful* deep ulcer w/ gray/yellow exudate >> Well demarcated borders & soft friable base > Severe lymphadenopathy that may suppurate Chlamydia > *Painless* Small shallow ulcers >> which can progress to painful, fluctuant adenitis (buboes) Syphilis > *Painless* Single ulcer (chancre) >> regular borders & hard base

A 39F, G1P0, at 34 wks presents for a routine prenatal visit. She has good fetal mvmts & some Lower abdominal pressure. Other than a cough & nasal congestion during the first trimester, the pt has had an uneventful pregnancy. Her first trimester screening & fetal anatomy US were both normal. She was Dx'd w/ HTN 5 years ago for which she takes labetalol. The pt drinks 2 cups of coffee daily but does not smoke, drink, of use drugs. She is a nurse in the neonatal ICU & plans to work until her due date. BP is 142/82 & fundal Ht is 30cm. US reveals a parietal diameter consistent w/ 32 wks & Abd circumference consistent w/ 27 wks. A normal amount of amniotic fluid & a posterior placenta are present. Estimated fetal wt is at the 8th percentile for gestational age. Which of the following is the most likely cause of the fetus' condition? (Caffeine consumption OR HTN)

HTN > Causes for asymmetric growth restriction are 1- Maternal malnutrition 2- Uteroplacental insufficiency (*HTN*, pregestational DM) >> even though the pt's HTN is controlled >> can still cause underdevelopment of the spiral arteries in the placenta, resulting in increased vascular resistance Caffeine consumption > Cocaine, Tobacco, and Alcohol all associated w/ FGR >> *not caffeine*

A 52F, presents d/t vulvar pruritus for the past wk. The pt also reports frequent urination, dysuria, & nocturia. She has had similar symptoms 4 times in the past 3 months. Each time previously, OTC suppositories have provided symptom relief. However, symptoms recur w/in the next 1-2 wks. The pt had irregular, heavy periods that improved after insertion of a progestin-containing intrauterine device. She has no other medical conditions or previous surgeries. Routine screening including colonoscopy, mammogram, and Pap test was normal 2 yrs ago. She is sexually active & in a long-term, monogamous relationship. The pt does not smoke, drink, or use drugs. BMI is 25. PE shows reveals vulvar erythema & multiple excoriations. There is thick, white discharge throughout the vaginal vault. No cervical or vaginal lesions are noted. Microscopy reveals pshudohyphae & no motile organisms. Which of the following additional testing should be performed in the pt? (Hb A1c OR NAAT)

Hb A1c > Pt presents w/ candidiasis >> RF include: 1- *DM* 2- Immunosuppression 3- Pregnancy 4- OCP's 5- Antibiotics NAAT > Used to Dx Chlamydia & nesseria

A 24F, presents d/t painful bumps in her goin. She has had these before, and the typically resolve w/in a few weeks, sometimes after draining spontaneously. Now the bumps occur more frequently & last longer, & sitting has become painful. The pt has no sig PMHx or prior surgeries. Temp is 98F, BMI is 35. PE shows multiple lesions w/ subsequent inflammation, thickened plaques, pitted acneiform scars, & fibrotic bands & keloid formation. What is the most likely Dx? (Hiradenitis suppurativa OR Lymphogranuloma venereum OR Granuloma inguinale)

Hiradenitis suppurativa > Chronic inflammatory condition that involves *recurrent* & * progressive* occlusions of hair follicles in intertriginous regions (groin, axilla) >> usually seen in *women 20-40* >> RF's include: 1- *Obesity* 2- Tobacco 3- FHx Lymphogranuloma venereum > Caused by Chlamydia >> Pts present w/ Painful inguinal lymphadenopathy (buboes) that can cause scarring & stricture w/in the inguinal region >> *However, does not cause recurrent infection* Granuloma inguinale > Caused by klebsiella >> typically presents w/ *painless, red ulcers* w/ no associated inguinal lymphadenopathy

A 65F, presents to a follow up for osteoporosis, which was Dx'd by a screening bone scan. The pt was prescribed alendronate, but she stopped taking it d/t intense stomach pain. She saw a TV ad about raloxifene & is interested in the TMT. The pt has a PMHx of DVT in her left leg while on an oral contraceptive at age 38 that was treated w/ several months of heparin. She currently takes meds for HTN & hyperlipidemia Dx'd after a minor heart attack at age 63. Her mom had breast cancer at age 52 & died from ovarian cancer at 61. A maternal aunt died from endometrial cancer at age 72. BP is 125/80, and PE is normal. Which of the following is a contraindication to raloxifene in this pt? (History of endometrial cancer in her maternal aunt OR Hx of venous thrombosis)

Hx of venous thrombosis > AE of Tamoxifen/Raloxifene are: 1- Hot flashes 2- *Venous thromboembolism* 3- Endometrial hyperplasia & carcinoma *Tamoxifen only* Hx of endometrial cancer > only if drug was tamoxifen

A pt was shown to have preeclampsia w/ severe features. During the pts eval, her BP increases to 170/115. Her pulse is 58/min. 15 mins later, her BP is 172/117. and pulse is 56/min. The pt has a severe headache & has an episode of emesis. Which of the following is the most appropriate med for obtaining bp control in this pt? (Methyldopa OR Hydralazine OR Labetalol)

Hydralazine > First line drugs for maternal HTN crisis are: 1- IV Hydralazine 2- IV Labetalol 3- Oral nifedipine Methyldopa > used to treat *chronic HTN* in pregnancy not HTN crisis Labetalol > also used in HTN crisis >> but pt has bradycardia already >> this will reduce HR even more

A 28F, primigravid, is admitted to the hospital at 10 wks. Her temp is 98.2F, pulse is 96/min, RR 12/min. Arterial blood gas shows - pH--------------7.49 - PaCO2---------54 - HCO3----------44 Which of the following is the most likely cause of this pt's abnormal arterial blood gas? (Hyperemesis gravidum OR Normal phenomenon of pregnancy)

Hyperemesis gravidum > using winters formula for alkalosis, expected PaCO2 is 53-57 >> so she has compensated metabolic alkalosis Normal phenomenon or pregnancy > Pt would have *hypocapnia* caused by hyperventilation

A 28F, nulligravid, presents for a routine exam. She feels well but has tried to conceive for the past 12 months w/out success. The pt has had sexual intercourse w/ her husband 3-4 times a week w/out contraception. Her menses are irregular with frequent missed periods. Every time the pts misses a period, her home pregnancy tests are negative. She has no medical conditions and has had no surgeries. The pt takes a prenatal vitamin & does not use tobacco, alcohol, or illicit drugs. Vitals are normal. There is no excess hair on PE. Breast exam shows no masses or nipple discharge. The abdomen is soft w/out masses. Pelvic exam shows normal external genitalia & a small, mobile uterus. Pelvic US shows a normal uterus & bilateral ovaries. Labs show: - Estradiol 7pg/mL (N>10) - FSH 3mIU/Ml (N>5) - LH 4mIU/mL (N>5) Which is the most likely Dx in this pt? (Hypogonadotropic hypogonadism OR Primary ovarian insufficiency)

Hypogonadotropic hypogonadism > Condition occurs when severe stress, excessive wt loss, or chronic illness cause a decrease in the amplitude & frequency of GnRH pulses secreted by hypothalamus >> leads to *low LH & FSH* >> Ovarian *estrogen production is low* >> ovulation does not occur Primary ovarian insufficiency >Typically presents w/ oligomenorrhea/amenorrhea, infertility, menopausal symptoms *Elevated FSH level + decreased estriol level* (pt has low FSH)

A 27F, G2P1, at 28 wks presents for follow-up of an abnormal pap. She feels fetal mvmt, has no vaginal bleeding or contractions, & has had an uneventful pregnancy to date. The pt's previous pregnancy 5 ys ago was uncomplicated. She has not had a previous abnormal Pap test, but the last test was performed during her prior pregnancy. The pt has no medical issues or previous surgeries. She takes a multivitamin & an iron supplement. She does not smoke, drink, or use drugs. BP is 120/74, Pulse is 82/min. Fetal HR are normal. PE reveals a gravid, nontender uterus. The cervix is long, closed, firm, & posterior, and the fetal presenting part is high. The Pap test showed a high-grade squamous intraepithelial lesion. Which of the following is the best next step in MGMT for pt? (Immediate colposcopy OR Repeat Pap test postpartum)

Immediate colposcopy > High grade squamous intraepithelial lesions (HSILs) are concerning for underlying severe neoplasia (CIN2, CIN3) >> *require immediate colposcopy* Repeat pap postpartum > Not appropriate >> *Pt presents w/ a HSIL ; must do immediate colposcopy*

A 13F, presents d/t lower abdominal pain for the past 4 days. The pt has rectal pain w/ bowel mvmts, but no melena or frank blood. She has had this pain several times over the past year, but usually resolves after 2-3 days. She has no fever, chills, nauseas, vomiting, loss of appetite, dysuria, hematuria, or abnormal discharge. The pt has no chronic med conditions or previous surgeries. She has not reached menarche & has never been sexually active. Temp is 98F, BP is 110/70, BMI is 24. PE shows a non distended abd w/out rebound or guarding. There is lower abdominal tenderness to deep palpation in the midline. The external genitalia are tanner stage 3. On PE, a smooth, firm mass is protruding between the labia majora. Which of the following is the most likely Dx? (Imperforate hymen OR Pelvic organ prolapse)

Imperforate hymen > Common anatomic cause of primary amenorrhea >> may present as: 1- *Blue, Bulging membrane/ vaginal mass* d/t mucus collection but this typically resolves & pts remain asymptomatic until menarche 2- Adolescents will present w/ *cyclic lower abd pain in absence of vaginal bleeding* Pelvic organ prolapse > May present w/ pelvic pressure & a vaginal bulge that increases w/ Valsalva >> Most common in postmenopausal women > *Not associated w/ primary amenorrhea*

A 25F, G2P1, at 8 wks presents to initiate prenatal care. Her blood type is O- & the fathers blood type is O+. Her first pregnancy was sig for placental abruption at the time of delivery. She received the standard dose of anti-D immune globulin at 28 wks during her first pregnancy & again 1 day postpartum. The pt has no medical problems & no Hx of blood transfusions. Her anti-D titer is currently 1:32. Which of the following is the most likely explanation for this pts finding? (Inadequate dose of anti-D after her first delivery OR Inadequate dose of anti-D during her first pregnancy)

Inadequate dose of Anti-D after her first delivery >> findings of 1:32 indicate that the pt was sensitized (alloimmunized) *mom who is Rh- had an Rh+ baby* >> to prevent maternal immune from developing anti-D antibodies she is given Anti-D globulin >> standard dose is 300ug at 28 wks >> *however ~50% of Rh- women will need a higher dose after delivery, placental abruption or other procedures* >> this can be calculated by the Kleihauer-Betke test Inadequate dose of Anti-D during first pregnancy > the standard dose at 28 wks in an uncomplicated pregnancy is usually adequate as the risk of alloimmunization is very low before this time

A 34F, G1P1, presents for an infertility eval. She has been trying to conceive for the past year, but her cycles has become increasingly irregular, w/ the last menstrual period more than 3 months ago. Menses previously occurred every 27 days & lasted 4 days. The pt feels fatigued & has been waking up at night d/t feeling warm. She has been married for 6 yrs & has. 4 yrs old daughter who was delivered vaginally w/out complications. The pt has hypothyroidism, for which she takes levothyroxine. She has no previous surgeries. The pt smokes a pack of ciggs daily but does not drink or use drugs. Both of her parents have DM II. BMI is 24, vitals are normal. PE shows normal external genitalia, a small mobile uterus, & normal bilateral ovaries. TSH is normal & a pregnancy test is neg. Which of the following would most likely be seen in this pt? (Decreased GnRH, Decreased FSH, Decreased estrogen OR Increased GnRH, increased FSH, decreased estrogen)

Increased GnRH, increased FSH, decreased Estrogen >Pt has *primary ovarian insufficiency* a form of *hypergonadotropic hypogonadism* >> *cessation of ovarian function <40 yrs* >> Characterized by 1- *Irregular menstrual periods* 2- *infertility* >> Pts usually have an *autoimmune disorder - Hypothyroid* or Turners Physio >> Decreased ovarian function results in *low estrogen levels* preventing negative feedback, so results in *increased GnRH & FSH* Decreased GnRH, Decreased FSH, Decreased Estrogen > Hypothalamic hypogonadism >> typically presents w/ 1- Oligomenorrhea OR Amenorrhea >> *but more likely in setting of caloric insufficiency* (eating/ exercise) >> *no associated menopause symptoms*

A 16F, presents d/y to recurrent, intermittent Lower abd pain. The pain typically begins the day prior to the onset of menses & subsides on the second day of bleeding. She has associated nausea & diarrhea during this time but had no constipation or pain w/ defecation. Theses symptoms began 5 months ago & have progressively worsened, causing the pt to miss 2 days of school last month. Menarche was at age 13 & her menses were irregular until 6 months ago; now menses occur every 28 days w/ 4-5 days of moderate bleeding. The pt recently became sexually active & uses condoms for contraception. Her last menstrual period was 2 wks ago. Vitals are normal. On speculum exam, a small amount of physiologic change is noted throughout the vaginal vault, & the cervix is nonfraible. The uterus is small, mobile, and nontender. Pregnancy test is neg. Which of the following is the most likely cause of this pt's pain? (Implanted endometrial glands outside the uterus OR Increased endometrial prostaglandin production)

Increased endometrial prostaglandin production > Pt presents w/ *primary dysmenorrhea* >> common in adolescents after the establishment of menstrual cycle >> this causes *increased endometrial prostaglandin production*, which causes uterine hypercontractility & hypertonicity >> resulting in ischemia >Classically manifests w/: 1- Cramping, midline lower abd pain that *begins 1-2 days prior to menses* & radiates to the lower back & thighs (Uterine hypertonicity & ischemia) 2- Systemic symptoms (Nausea, vomiting, diarrhea d/t PG induced stimulation of GI tract) Implanted endometrial glands outside the uterus > Would cause dysmenorrhea >> However, Pt's typically have *pelvic pain throughout menses* (Pt's pain resolves 1-2 days after onset) >> or pain that may persist beyond menses > Would not be associated w/ GI issues

A 36F, G2P1, at 9 wks presents to initiate prenatal care. The pt's first pregnancy ended in a vaginal delivery after an induction for preeclampsia at 38 wks. After her first pregnancy, the pt had orthopedic surgery for an ankle fracture that was complicated by a DVT, which was treated w/ anticoagulation. She has otherwise been healthy & active. She is taking prenatal vitamins & no other meds. BP is 120/70, Wt is 121.3 lbs. PE shows an enlarged contender uterus w/ no adnexal masses. D/t the pt's Hx of preeclampsia & DVT, a CBC, basic metabolic panel, hepatic function panel, Thrombophilia panel, & urinalysis & urine culture are ordered. Which of the following changes may be seen in a healthy pregnant pt when compared to the pre-pregnant state? (Decreased serum creatinine OR Increased platelets)

Increased serum creatinine > In normal pregnancy: 1- IncreasedGFR 2- Increased RBF 3- Increased BM permeability >> all are increased which causes the *serum BUN & Creatinine to become decreased* compared to pre-pregnancy levels Increased platelets > Although there is a hyper coagulable state in pregnancy it is *not caused by increased platelets* >> is is caused by changes in hemostatic & fibrinolytic proteins w/ 1- decreased protein S activity 2- Increase in fibrinogen 3- Increased resistance to activated protein C

A 31F, at 7 wks, presents to the ED by her husband d/t vaginal bleeding & lower abd pain. The bleeding began 4 hrs earlier & the pt has saturated 3 perineal pads in that time. Her abdominal cramping has become progressively more severe & unresponsive to acetaminophen. She has no fevers, chills, of abnormal vaginal discharge. The pt was treated at age 18 for Chlamydia cervicitis. Her surgical Hx includes a loop electrode excision procedure for high-grade cervical intraepithelial neoplasia, but she has had a normal Pap test since. The pt takes a prenatal vitamin daily & does not smoke, drink, or use drugs. Temp is 98.7F, BP is 100/76, Pulse is 112/min, & RR is 14/min. PE reveals blood clots in the vaginal vault & active bleeding from a dilated cervix. A bimanual exam demonstrates a 6 wk sized tender uterus. US reveals a gestational sac in the lower segment of the uterus, a simple cyst on the right over, & free fluid in the posterior cup-de-sac. Which of the following is the most likely Dx? (Inevitable abortion OR Missed abortion OR Threatened abortion)

Inevitable abortion > Pt has: 1- Pain 2- Vaginal bleeding 3- Dilated cervix 4- Products of conception in the lower uterine segment Missed abortion > pt would not have bleeding and would have closed cervix Threatened > pt would have bleeding but *closed cervix* > No clots (products of conception) would be seen > Fetal heart activity would be present

A 42F, gravida 3 para 3, presents to the office d/t to left breast swelling & pain that has worsened over the past month. The pt weaned her youngest child from breastfeeding approximately 2 months ago. She last saw a health care provider a month ago for mastitis & was prescribed antibiotics. BMI is 46.6. Temp is 99.7F & BP is 110/68. PE reveals a morbidly obese woman in no apparent distress. Breast exam shows a left breast that is diffusely warm & erythematous w/ some dimpling. The right breast appears normal. Which of the following is the most likely Dx? (Breast Abscess OR Inflammatory breast carcinoma)

Inflammatory breast carcinoma >Usually presents as rapid-onset edematous cutaneous thickening w/ a "peau d'orange" appearance (*superficial dimpling*, fine pitting) > affected breast is *diffusively edematous, erythematous, & painful* Breast Abscess > can be confused w/ an infection >> however if was infection it would've improved with antibiotics > Would not have dimpling > may have discharge > Would present as a *Localized, painful mass* (whole Breast would not be affected)

A 25F, gravida 2 para 1, at 24 wks presents for a routine prenatal visit. She has felt normal fetal mvmt & no abd pain, vaginal bleeding, or leakage of fluid. Three days ago, the pts 5 yr old son was diagnosed w/ varicella.; however, she has not had fever, malaise, or a rash. The pt has no chronic medical issues or previous surgeries. She takes a prenatal vitamin & iron supplement. The pt does not use tobacco, alcohol, or illicit drugs. BP is 110/60 And pulse is 64/min. BMI is 27. PE shows no rash of lymphadenopathy. Fundal Ht is appropriate for age. Fetal HR by bedside Doppler is 145/min. Prenatal labs show: - Rubella IgG neg - Rubella IgM. neg - Varicella IgG. pos - Varicella IgM. neg Administration of which of the following is the best next step for this pt? (MMR OR Influenza virus vaccination)

Influenza virus vaccination >Recommended vaccinations during pregnancy are: 1- Tdap (toxoids & inactivated vaccines) 2- Inactivated influenza 3- Rho (D) MMR - MMR is CI in pregnancy

A 18F, nulliparous, presents w/ lower abdominal pain, nausea, & vomiting for the past day. She is unable to keep anything down. The pt is sexually active & uses oral contraceptives. She has had 3 lifetime sexual partners. Her last menstrual period was 15 days ago. She has not received the HPV vaccine. The pt has no allergies & takes no meds. Temp is 102.2F, BP is 100/70, Pulse is 110/min. PE reveals dry mucous membranes. The abdomen has diffuse tenderness over the lower quadrants. External genitalia shows no abnormalities; speculum exam reveals purulent discharge from the cervical os. The uterus is small, anteverted, and tender to palpation & motion. The adnexae are markedly tender bilaterally w/ no palpable masses. Cervical nucleic acid amplification testing is pending. Urine pregnancy test is negative. Labs show - Leukocytes---------18K - Neutros-------------80% Which of the following is the most appropriate next step in MGMT? (Inpatient TMT w/ cefotetan + doxy OR Outpatient TMT w/ ceftriaxone + doxy)

Inpatient TMT w/ cefotetan + doxy > Pt has *PID* >> which can present w/: 1- *Fever* 2- *Lower abd pain* 3- *Purulent cervical discharge* 4- *Cervical motion tenderness* >> usually caused by chlamydia + nesseria TMT >> hospitalization & parenteral antibiotics d/t persistent nausea, vomiting, dehydration, *severe presentation* (high fever, leukocytosis) - *IV cefoxitin or cefotetan plus doxy* - *Parenteral clindamycin + gentamicin* Outpatient TMT w/ ceftriaxone + doxy > Would be correct if pt was stable *and did not have severe presentation*

A 15F, presents d/t irregular menstrual periods. Menarche was at age 13, & since then her periods have been irregular w/ cycles varying from 3-8 wks. The pt has no chronic medical issues, has never had surgery, and takes no meds. Her mother & sis have PCOS, & both take OCP's. The pt has never been sexually active. Vitals are normal. PE shows normal hair distribution w/ Tanner V secondary sexual characteristics. Abd exam is normal. There is dark red blood noted at the cervical os w/ no active bleeding. Serum prolactin & TSH levels are normal. Administration of micronized oral progesterone results in withdrawal bleeding in 3 days. Pelvic US reveals normal ovaries & uterus. Which of the following is the most likely explanation of this pts' irregular menstrual periods? (Insufficient gonadotropin secretion OR Intrauterine adhesions)

Insufficient gonadotropin secretion > Initial menstruation cycles in adolescents are irregular & anovulatory >> Abnormal uterine bleeding that follows menarche is d/t *immaturity of the developing HPG axis that produces inadequate amounts of GnRH* >> leading to low FSH & LH >> leading to an absence on ovulation Intrauterine adhesions > pt would have history of some procedure (which she does not) or infection (endometritis) > She would not have bleeding after progestin withdrawal challenge d/t lack of endometrial tissue

A 36 F, nulligravida, presents for an infertility eval. Eight months ago, she was found to have a submucosal fibroid & underwent uncomplicated hysteroscopic resection. The pt had light spotting for several days after the surgery but no fever or abnormal vaginal discharge. Her last menstrual period was immediately prior to the procedure , but now she has monthly pelvic pain w/out bleeding. The pt has had regular, unprotected sex for the last 6 months w/out conception. She has no chronic medical conditions & has had no other surgeries. Vitals are normal. The uterus is small, mobile, and non-tender. Pregnancy test is negative. FSH, TSH, & prolactin levels are normal. STD screening is negative. A progesterone withdrawal test does not induce vaginal bleeding. Which of the following is the most likely cause of this pts infertility? (Intrauterine synechiae OR Tubal occlusion)

Intrauterine synechiae > This pts infertility & secondary amenorrhea after hysteroscopic myomectomy is likely d/t *Asherman syndrome* (the development of symptomatic intrauterine synechiae >> Pts who undergo intrauterine surgery are at risk Clinical features include - Abnormal uterine bleeding - Amenorrhea - Infertility - Cyclic pelvic pain - Recurrent pregnancy loss Tubal occlusion > Pt would present w/ signs of *PID/endometriosis* > PID ---Cervical motion tenderness (Chandelier sign >> jumping off table) ---Vaginal bleeding ---Foul smelling discharge ---Lower Abd pain (2ndary to uterine and/or adnexal tenderness) *w/ notable tenderness on bimanual exam* ---GI distress, Diarrhea ---Painful urination Endometriosis - Pain at other times rather than menses - Increased bleeding during times of menses - Dysmenorrhea, Dyschezia, Dyspareunia

A 2F, presents w/ an abnormality in the genital area seen while changing diapers. The pt has had multiple diaper rashes over the last 6 months but has had regular bowel mvmts & no dysuria or hematuria. She has no Chronic medical conditions & no known allergies. Vitals are normal. On pelvic exam, the labia minora appear thin, are fused together at the midline, and partially block the urethral meatus. Multiple excoriations are seen, but there is no rash of perianal involvement. No vaginal discharge is seen at the Introitsus. Internal pelvic exam is deferred. Which of the following is most likely Dx? And what is the fist line TMT? (Labial adhesions OR Lichen sclerosis)

Labial adhesions > Thin, Fused labia minora d/t labial adhesions >> most commonly seen in prepubertal girls d/t *low estrogen production* TMT > topical estrogen (first line) Lichen sclerosis > Type of skin inflammation (also contact dermatitis) which can predispose to labial adhesions >> However presentation for each is: - Lichen sclerosis >> hypopigmented labial lesions - Contact dermatitis >> Pruritic erythematous rash

A 24F, G2P1 at 28 wks presents w/ her husband b/c she has felt no fetal mvmts for 3 days. She has no pain, leakage of fluid, or bleeding. All testing during this pregnancy has been normal, including an anatomy US at 20 wks. Her first child was delivered vaginally at term. Temp is 98F, BP is 120/80, Pulse is 94/min. Fetal heart tones are not heard by doppler. US demonstrated an absence of fetal cardiac activity & a small for gestational age fetus in breech presentation. A digital cervical exam shows the cervix to be long & closed. Labs are normal. Which of the following is the most appropriate MGMT advice for this pt? (Dilation & evacuation under general anesthesia OR Labor can be induced when you feel ready)

Labor can be induced when you feel ready > *Vaginal delivery is preferred at >/= 24 wks regardless of fetal presentation* >> retention of the fetus for several wks can lead to coagulopathy Dilation & evacuation under general anesthesia > indicated for an IFUD that is Dx'd at *< 24 weeks* (pt is at 28 weeks)

A 25F, nulligravid, presents d/t pelvic & lower sacral back pain. The pain has worsened over the past year, intensifies a few days before menstruation, & improves moderately toward the end of her period. Ibuprofen only minimally diminishes the pain. She has no fever or abnormal vaginal discharge. The pt has been in a monogamous relationship w/ her bf for the past 4 yrs. For the first 3.5 years, they used condoms for contraception; she began taking oral contraceptives 6 months ago. Temp is 98.9F, BP is 120/78. Exam shows tenderness in the posterior vaginal fornix, decreased uterine mobility, & thickening of the uterosacral ligaments. No adnexal masses are palpated. Urine B-hCG is negative. Hb is 12 g/dL & leukocyte count is 8,2K. Transvaginal US shows normal pelvic c anatomy. Which of the following is the most appropriate diagnostic test for this pt? (Hysterosalpingogram OR Laparoscopy)

Laparoscopy > Pt has endometriosis >> can present w/: 1- Dysmenorrhea 2- Dyspareunia 3- *Chronic pelvic pain that worsens before onset of menses* >Exam findings 1- Tenderness of the recto-vaginal area 2- *Tenderness w/ mvmt of the uterus* 3- *Thickening of the uterosacral ligaments* caused bu endometrial implants on the recto-vaginal septum, pelvic or peritoneum, anterior & posterior cul-de-sac, & uterosacral ligaments *Laparoscopy is indicated* after failure of empiric therapy (NSIADs or oral contraceptives) Hysterosalpingogram > A diagnostic test to check the shape of the uterine cavity & Fallopian tube potency >> *commonly used for infertility*

A 19F, presents to the ER d/t lower abdominal pain. The pain is intermittent & started this morning during a yoga class. Over the past 10 hrs, the pain has intensified & ibuprofen has provided no relief. The pt has also vomited 3 times. Her menstrual period was a week ago. She has never been sexually active. The pt has no medical conditions and no history of surgery. She takes no meds and does not smoke, drink, or use drugs. PE shows diffuse lower abd tenderness, left greater than right, w/out rebounding or guarding. Pelvic US shows a complex left adnexal mass w/out Doppler flow. There is a small amount of free fluid. Which of the following is the best next step in MGMT? (Laparoscopy OR XR of abdomen

Laparoscopy > Pt is presenting w/ signs of ovarian torsion >> signs are: 1- Sudden-onset Unilateral lower abd pain (which may vary from sharp to colicky intermittent pain) 2- Nausea and/or vomiting > *can be precipitated by exercise* Pelvic US findings > *Pathognomonic Adnexal mass w/ lack of doppler flow* TMT > *Laparoscopic cystectomy & detorsion >> goal is to restore normal anatomy & save the ovary from irreversible necrosis XR of abdomen > XR of abdomen would be used to look for *bowel obstruction* >> Pt would present w/ many of the same symptoms >> but RF's include: 1- Advanced peritoneal/ ovarian cancers 2- Adhesions from multiple previous surgeries >> unlikely in this pt b/c lack of surgery in history and young age

A 32F, presents w/ pelvic pain for the past 2 days after her menstrual period ended. The pt says that the pain "feels like labor contractions" & ibuprofen provides no relief. She has a history of regular but heavy menstrual periods & occasionally passes clots. The pt is sexually active & uses the rhythm method for contraception. 5 yrs ago, she had a spontaneous abortion treated by suction curettage & dilation. Pap test was normal 2 yrs ago. BP is 140/90, & pulse is 113/min. PE shows an irregularly enlarged uterus. The cervix is dilated 5 cm w/ a spherical mass visible through the external os. The mass is firm & smooth, & slight bleeding is noted around it. Which of the following is the most likely Dx? (Cystocele OR Leiomyoma uteri)

Leiomyoma uteri > Presents w/ *Irregularly enlarged uterus* >> pt has typical presentation of prolapsing leiomyoma uteri >> Speculum and bimanual exam would show the firm, smooth, round mass at the cervical os consistent w/ an aborting submucous myoma >> most common symptom is 1- *Heavy & prolonged menstrual bleeding* 2- pts can also present w/ Labor-like pain d/t to mechanical cervical dilation by the solid mass & is preceded by heavy vaginal bleeding Cystocele > (bladder prolapse) >> characterized by pelvic pressure, urinary symptoms, and/or a palpable vaginal bulge *not heavy menses* > cervix would not have any masses

A 32F, G4P4, presents for a routine check up 6 wks after an uncomplicated vaginal delivery of a healthy baby boy. The pt has no concerns & would like to discuss contraception options. She heard from a friend that some options make losing weight challenging. Prior to this pregnancy, the pt had a history of increasingly heavy menses & was found to be mildly anemic. Iron was prescribed, but she often forgot to take it; she often forgot her prenatal vitamins too. The pt is breastfeeding exclusively. Pelvic exam shows a small, mobile uterus w/ no abnormalities. Breast exam shows no masses, tenderness, or nipple injury. Which of the following is the preferred method of contraception for this pt? (Levonorgestrel containing intrauterine device OR Medroxyprogesterone injection)

Levonorgestrel containing IUD > It is a long acting, reversible contraceptive that prevents pregnancy by releasing progestin >> AE include: 1- Mood changes 2- Breast tenderness 3- Headaches *not weight gain* Medroxyprogesterone injection > Option for long-term contraception >> pts receive injections every 3 months >> *weight gain is common side affect*

An 11F, presents d/t vulvar itching. The pt first had the pruritus intermittently, but her symptoms have worsened over the past several months despite nightly application of a moisturizing ointment. She now has trouble sleeping d/t to constant itching. The pt has no medical conditions & has had no surgeries. She has not reached menarche & is not sexually active. The pt is on a community swim team & has swim practice multiple times a week & uses the same bathing suit. Vitals are normal. Exam of the vulva shows thin, white skin w/ excoriations extending to the perianal area. The superior portions of the labia minora are adherent to the midline. A small anal fissure is also noted. Which is the most likely dx? (Labial adhesions OR Lichen sclerosis) And what is the TMT?

Lichen sclerosis > Prominent in hypoestrogenic girls (such as *premenarchial* & postmenopausal women) Presents w/: 1- *Thin, white, wrinkled skin over the labia minora/majora* 2- changes that may *extend into the perineum & around the anus* 3- *Excoriations, erosions, fissures from severe pruritus* 4- Dysuria, dyspareunia, painful defecation TMT > superpotent topical corticosteroids *Clobetasol* Labial adhesions > Can cause vulvar pruritus d/t fusion of labia at midline >> however *most common in girls <2* > not associated w/ lichenification & no perianal involvement

A 54F, G2P2, presents for eval of leakage of urine. Over the past 2 yrs, the pt has had increasingly frequent involuntary loss of urine & now wears a sanitary napkin to prevent urine from dribbling on her clothes after she voids. For the past week, she has had burning w/ urination but no frequency or urgency. In the last year, the pt has had 3 UTI that were treated w/ antibiotics. The pt had 2 term vaginal deliveries in her 30s. She has no chronic medical conditions & has had no prior surgeries. BP is 118/64, Pulse is 82/min. BMI is 33. On pelvic exam, the pt does not leak urine w/ Valsalva & the vagina is dry & atrophic. A 2-cm tender anterior vaginal mass is palpable & causes expression of a bloody discharge at the urethral meatus. Postvoid residue is 50mL. Which of the following is the most likely causes of the pt's symptoms? (Abnormal tract between the bladder & vagina OR Localized out pouching of urethral mucosa)

Localized out pouching of urethral mucosa > Pt has a *urethral diverticulum* >> an abnormal localized urethral mucosa d/t recurrent periurethral gland infection along the anterior vaginal wall >> Recurrent infection creates an abnormal out pouching that can collect & store urine, resulting in *postvoid dribbling & recurrent lower UTI's* >> infection of the urethral diverticulum can also result in: 1- Tenderness, often presenting as dyspareunia or a *tender anterior vaginal wall mass w/ a purulent or bloody urethral discharge* Abnormal tract between the bladder & vagina > This is called a vesicovaginal fistula >> would present w/ constant (rather than intermittent) leakage of urine d/t continuous drainage of the bladder in to the vagina

A 55F, presets d/t occasional bloody spotting on her toilet paper over the past few weeks. The pt has no hematuria, dysuria, hematochezia, melena, heavy vaginal bleeding, or vulvar pruritus. Last menstrual period was 4 yrs ago. She had a normal Pap last year w/ no Hx of abnormal results. She had no medical conditions & takes daily supplements of calcium w/ vitamin D. The pt is not sexually active & does not smoke, drink, or use drugs. BMI is 32. PE shows sparse pubic hair & fissures along the vestibule. The vagina has multiple areas of petechiae; the cervix is flush w/ the vaginal wall. There is minimal clear vaginal discharge but no active bleeding. Vaginal pH is 6. Urinalysis is normal. Fecal occult blood test is negative. Pelvic US reveals a small, anteverted uterus w/ a 3-mm endometrial stripe. Which of the followings is the most likely explanation for the findings in this pt? (Loss of vaginal elasticity OR Vaginal intraepithelial neoplasia)

Loss of vaginal elasticity > Pt presents w/ genitourinary syndrome of menopause >> which presents w/: 1- Vulvovaginal dryness, irritation, pruritus *petechiae* 2- Dyspareunia 3- *Vaginal bleeding* 4- Urinary incontinence, recurrent UTI 5- Pelvic pressure 6- *pH >/= 5* 7- *sparse pubic hair* Physio *D/t loss of epithelial elasticity & subsequent atrophy* >> *seen in pt by cervix being flush w/ vaginal wall* Vaginal Intraepithelial neoplasia > would present w/ similar symptoms (abnormal vaginal bleeding & discharge) >> however Typically associated w/ HPV (pts last Pap was normal) > pt would have concurrent cervical dysplasia

A 27F, primigravid at 16 wks presents for an initial prenatal visit. She has had no pelvic pain, vaginal bleeding, or abnormal discharge. The pt immigrated to the US 6 years ago. She has no medial issues or previous surgeries. She has had regular menses prior to pregnancy & has no Hx of STD's. FH is noncontributory. The pt takes a daily prenatal vitamin & does not smoke, drink, of use drugs. BP is 110/70, BMI is 23. Fetal heart rate is 155/min. The pt's blood type is O, Rh-. Rubella and varicella titers both show no immunity. Hb is 11.2 and MCV is 84fL. All other routine prenatal labs are normal. Which of the following is the best recommendation for this pt? (MMR vaccine postpartum OR Rho(D) Ig now)

MMR postpartum > All rubella-nonimmune pts should be vaccinated during the immediate postpartum period Rho (D) Ig now > Administration is recommend *at 28 wks* gestation for prophylaxis & after delivery if the infant is Rh(D) + >> *first trimester administration is justified only in the setting of uterine bleeding*

A 36F, G1P0, at 36 wks is undergoing induction of labor d/t preeclampsia w/ severe features. The pt is receiving a magnesium sulfate infusion for seizure prophylaxis & an oxytocin infusion for labor augmentation. She had spontaneous ROM w/ clear fluid 4 hrs ago. The pt had an epidural anesthesia placed d/t increasingly painful contractions & now has adequate pain relief. She has a hx of hypothyroidism that has been poorly controlled during pregnancy. Temp is 103F, BP is 150/90, pulse is 114/min. Fetal heart tracing shows Tachycardia (Baseline at 180). Which of the following is the most likely cause of the pt's fetal heart rate tracing? (Maternal fever OR Magnesium sulfate)

Maternal fever > causes tachy, Other causes are: 1- Med AE (beta agonists) 2- Fetal hyperthyroidism 3- Fetal tachyarrhythmia Magnesium sulfate > Fetal heart tracings typically reveal a transient decrease in variability d/t CNS depression

A 33F, presents for eval of "fluid coming out of both nipples." The fluid is pale grey & has been leaking intermittently for the past week. She has 2 healthy children ages 5 & 8. The pt had a copper IUD a few months after the delivery of her younger child & has not used any hormonal OCPs. She also has a Hx of mild GERD for which she takes OTC antacids. Exam shows normal breasts w/out palpable lumps or nipple abnormalities. Brownish-gray discharge is expressed from both nipples and is guaiac neg. Urine pregnancy test is neg. Which of the following is the most appropriate next step in MGMT? (Measure prolactin & TSH levels OR Mammography)

Measure prolactin and FSH levels > Pt presents w/ physiologic galactorrhea >> which presents w/ *Bilateral, gray, non-bloody nipple discharge* >> *hyperprolactinemia* is most common cause Mammography > If pt presented w/ signs suggesting malignancy (unilateral discharge, serous or bloody rather than milky, palpable breast abnormalities, & associated skin changes >> then we would use mammogram

A 37F, G2P1, at 8 wks presents for a follow-up visit. Earlier in the pregnancy, the pt had nausea & breast tenderness that resolved several days ago. She has no other concerns. At her initial prenatal visit a week ago, US revealed an intrauterine gestational sac w/ a yolk sac but no fetal pole. B-hCG level at that visit was 27,325. She is takin a prenatal vitamin & does not drink, smoke, or use drugs. Current BP is 140/80, pulse is 68/min. BMI is 23. Pelvic exam reveals a closed cervix & no vaginal discharge or bleeding. Bimanual exam reveals a normal sized, retroverted uterus w/ no cervical motion or adnexal tenderness. A transvaginal US is repeated & is unchanged from the prior visit. B-hCG level is now 25,659. Which of the following is the most likely Dx? (Hydatidiform mole OR Incomplete abortion OR Missed abortion OR Threatened abortion OR Inevitable abortion)

Missed abortion > An intrauterine demise at <20wks gestation >> presents w/: 1- *No vaginal bleeding* 2- *Closed cervix* 3- No fetal cardiac activity 4- Empty sac *Intrauterine sac w/ no fetal pole* Threatened abortion 1- Vaginal bleeding 2- Closed Cervical os 3- Fetal cardiac activity Inevitable abortion 1- Vaginal bleeding 2- *Dilated Cervical os* 3- Products of conception may be seen or felt at or above the cervical os Incomplete abortion 1- Vaginal bleeding 2- Dilated cervical os 3- Some products of conception expelled & some remain Hydatidiform mole > would present w/: 1- Heavy bleeding (pt has none) 2- Snowstorm appearance on US 3- Markedly elevated B-hCG

A 23F, primigravid, at 29 wks presents to the ER w/ sudden-onset severe SOB. She woke up today w/ a fluttering sensation in her chest, which progressed to significant dyspnea, dry cough, & an inability to lie flat. Recently, the pt has had some exercise intolerance & tiredness, which she attributes to pregnancy. Her medical Hx is sig for recurrent sore throats requiring tonsillectomy as a child. She is a lifelong nonsmoker & does not drink. The pt's lifestyle is mostly sedentary. She immigrated to the US from India 5 yr ago. BP is 110/60, Pulse is 144/min. The pt appears uncomfortable. An ECG shows A-fib w/ rapid ventricular response. Which of the following is the most likely Dx? (Mitral stenosis OR Peripartum cardiomyopathy)

Mitral stenosis > Pt's PMHx most likely means Rheumatic fever >> during pregnancy, the physiologic changes increase HR and Blood volume raising the trans-mitral gradient & left atrial pressure (at rest & during exercise) >> this presents w/ symptoms: 1- Fatigue 2- Exercise intolerance 3- Dyspnea >> in Rheumatic fever pts that were previously asymptomatic Peripartum cardiomyopathy > Causes rapid onset systolic heart failure (fatigue, dyspnea, cough, pedal edema) *at > 36 wks* >> this pt presents at 29 wks and has history of RF

A 16F, presents to the ER w/ lower left quadrant pain that suddenly started 24 hrs ago. The pain does not radiate & is 5/10 in severity. The to does not report fever, vomiting, dysuria, diarrhea, or vaginal bleeding. Her last menstrual period was 2 wks ago. Menses occur regularly every 28 days & last 5 days w/ cramping only on the first day. She takes no meds. Temp is 98.8F, BP is 110/65, Pulse is 80/min. PE shows mid left lower quadrant tenderness w/out rebound. Which of the following is the most likely Dx? (Mittelschmerz OR Ovarian hyperstimulation syndrome)

Mittelschmerz > this is pain in the middle of the menstrual cycle caused by normal ovulation >> typically occurs 10-14 days from first day of previous menses >> pts may have a recurrence of monthly instance of pain halfway thru their cycle or *experience a single more uncomfortable episode* > *pain is usually unilateral* and last *less than a day* >> resolves w/out intervention Ovarian hyperstimulation syndrome > Iatrogenic complication of ovulation-inducing meds >> characterized by *diffuse abd pain* d/t ovarian enlargement by multiple follicles & may be accompanied by *ascites & respiratory difficulties* - all absent in this pt

A 23F, primigravid, at 20 wks, presents for routine prenatal visit. She has no vaginal bleeding, leakage of fluid, or contractions. The pt had a mild nausea that resolved at 14 wks gestation. A fetal anatomic sonogram performed a week ago revealed no abnormalities. The pt has no chronic medical issues or previous surgeries. She is taking a daily prenatal vitamin & does not smoke, drink, or use drugs. The pt works in a lab as a bio-tech RA. She has Blood group O, Rh+, & her husband has AB, Rh+. On PE, the fundus is palpable at the umbilicus, and the fetal HR is 140/min. The pt is concerned about diff blood groups & the risk of pregnancy complications. Which of the following is the most appropriate response to this pt? (Antibodies to ABO Ag's are not hemolytic OR Most newborns affected by ABO incompatibility will only have mild hemolytic disease OR You are tolerant to your child's ABO Ag's, so your newborn will not have hemolytic disease)

Most newborns affected by ABO Ag's incompatibility will only have mild hemolytic disease > Typically mild & may be asymptomatic >> usually expressed in only 1/3 of infants >> may develop some jaundice w/in first 24 hours of life which responds to phototherapy you are tolerant to your Childs ABO Ag's, so your newborn will not have hemolytic disease >Exposure to the A & B Ag's of the fetus does provoke an IgM response, but the antibodies cannot cross the placenta Antibodies to ABO Ag's are not hemolytic > Antibodies are hemolytic >> However, in addition to RBC, A & B antigens are presents of the cells of all other fetal tissues >> which leads to mild response

A 32F, G2P1, at 28 wks presents for routine prenatal visit. 3 wks ago, she developed a facial rash that has darkened despite the use of lotions. For the past wk, the pt has been unable to work at her job at a landscaping company d/t increasing fatigue & dizziness on standing. She has had no contractions, vaginal bleeding, or leakage of fluid. Fetal mvmt is normal. The pt has no chronic medical conditions & takes a prenatal vitamin & iron supplement daily. Temp is 98.1F, BP is 110/70, Pulse is 72/min. Fetal Heart rate is 145/min. Light brown, irregularly shaped macules are arranged in bilateral symmetric fashion on the pt's cheeks & nasal bridge. The macule do not affect the nasolabial fold. Labs are normal. which of the following is the best next step in eval of this pt? (Antinuclear antibody testing OR No further eval needed)

No further eval needed > Pt has *melasma* >> acquired hyperpigmentation disorder that occurs primarily on sun-exposed areas of the face >> More common in *Women of reproductive age during pregnancy* Antinuclear antibody testing > Seen in SLE; pt would have previous history >> *rash is typically erythematous & scaly appearing* >> pt would also have 1- HTN 2- Anemia 3- Thrombocytopenia 4- Proteinuria

A 67F, G1P1, presents for a routine follow up. She is feeling well. Her last menstrual period was at age 53 w/ no bleeding since then. Pap test & HPV co-testing were negative 2 yrs ago at age 65. All prior Paps were normal. The pt is sexually active in a monogamous relationship w/ her husband of 42 years. She has no medical conditions. She had an ovarian cystectomy via laparotomy at age 37 for a benign cystic teratoma. The pt takes no meds & does not smoke, drink, or do drugs. There is no FHx of cancer. BMI is 25. Pelvic exam shows a normal cervix w/out lesions; a smooth, anteverted, mobile uterus; & no adnexal masses. Screening mammography was normal 6 months ago, & screening colonoscopy was normal 2 years ago. Which of the following is the most appropriate recommendation for cervical cancer screening in this pt? (No further screening OR Repeat Pap test in 3 yrs)

No further screening > When to Stop Pap testing guidelines 1- Age 65 or hysterectomy *PLUS* No Hx of CIN 2 or higher *AND* 3 consecutive negative Pap tests *OR* 2 consecutive neg co-testing results Repeat Pap test in 3 yrs > Pt meets guidelines to stop Pap testing (above)

A 58F, nulliparous, presents for a routine visit. She is concerned about ovarian cancer b/c her cousin died of the disease last year & a close friend was recently Dx'd with it. PMH is sig for HTN treated w/ an ACEI. Menopause was at age 55. The pt smokes half a pack a day & consumes alcohol occasionally. There is no other cancer Hx in her family. PE is unremarkable. She has never had an abnormal Pap result, & the last tests was 2 yrs ago. Which of the following is the most appropriate strategy for ovarian cancer screening in this pt? (No further work-up OR Serum CA-125 level)

No further work-up > No screening tests exist to detect ovarian cancer in its early, more treatable stages >> (The pt's FHx of a single second-degree relative w/ ovarian cancer does *NOT increase the likelihood* she will get cancer Serum CA-125 levels > CA-12 can be used to: 1- Help differentiate a malignant Vs benign ovarian mass 2- False elevation d/t endometriosis/ leiomyomata

A 32F, is evaluated at 32 wks. Labs show: - Arterial pH-----7.45 - PaO2------------110 - PaCO2----------30 - Bicarb ----------20 -Hb---------------11g/dL - WBC-------------9K - Na+--------------134 - K+----------------3.6 - Cl---------------- 102 - BUN--------------5mg/dL - Creatinine--------0.6mg/dL Which of the following best explains this pts acid-base status? (Normal pregnancy OR Preeclampsia)

Normal pregnancy > Pts have exaggerated respiratory effort >> hyperventilation (alkalosis-pH + low PaCO2) Preeclampsia > Commonly would present after 20wks > would have proteinuria or end organ damage (not seen in pt)

What is the most appropriate TMT for the pt in previous question? With the Bartholin duct cyst? (Incision & drainage OR Observation & expectant mgmt?

Observation and expectant mgmt > Asymptomatic cysts *do not require intervention* as most drain spontaneously & resolve on their own Incision & drainage > If pt had a symptomatic cyst or abscess >> then would drain

A 2hr-old-boy is evaluated in the delivery room for minimal right arm mvmt. He was born at 37 wks by vacuum assisted, vaginal delivery to a 39F who had excessive wt gain during pregnancy. The delivery was complicated by a 3-min shoulder dystocia. Birth wt was 4.6kg (10.1 lbs). Exam shows a small area of edema in the scalp that does not cross suture lines. The RUE is held in adduction & internal rotation w/ the elbow extended, forearm pronated, & wrist & fingers flexed. Moro & biceps reflexes are absent in the right arm. The remainder of the exam is normal. Which of the following is the most appropriate next step in mgmt for this pt? (MRI of spine OR Observation only)

Observation only > MGMT involves observation & PT b/c 80% of pts have spontaneous recovery w/in 3 months >> prognosis depends on whether the damage resulted from mild nerve stretching or compression as opposed to severe rupture or avulsion MRI of spine > It pt's symptoms do not spontaneously improve >> MRI of the cervical spine may be used to detect proximal nerve root avulsion which requires surgery

A 32F, G3P2, at 41 wks presents for a routine prenatal visit. She has had no contractions, vaginal bleeding, or leakage of fluid. Fetal mvmt is normal. The pt's pregnancy has been uncomplicated. An US at 18 wks showed normal fetal anatomy & a posterior placenta. She has no chronic medical conditions & has had no previous surgeries. The pt takes a daily prenatal vitamin & does not use tobacco, alcohol, or drugs. BP is 110/80, Pulse is 80/min, BMI is 28. Fundal Ht is 40cm. The cervix is 1 cm dilated & 10% effaced. NST is reactive. The pt would like to await spontaneous labor rather than undergo an induction. Delaying delivery to 42 wks in the pt increases the risk of which of the following pregnancy related complications? (Low fetal birth weight OR Oligohydraminos)

Oligohydraminos > Delaying in post term pregnancy can result in any of these complications: *Fetal/neonatal* 1- Macrosomia 2- Dysmaturity syndrome 3- *Oligohydraminos* 4- Demise *Maternal* 1- Severe obstetric laceration 2- CS 3- Postpartum hemorrhage Low fetal birth weight > Typically *seen in preterm births* >> Macrosomia seen in post term births

A 33F, G2P1, Presents for routine prenatal visit at 26 wks gestation. Her pregnancy has been uncomplicated. The pts BMI was 23 at her initial prenatal visit, & her lab results were as follows: - Hct ---------34% - HBsAg. -----neg - HIV-1 Ab ---neg - RPR. --------neg - ABO/Rh. ----A+ - Antibody ---neg - Urine. neg -Chlamydia neg An US at 20 wks showed an anterior placenta & normal fetal anatomy. The pt's only concern today is lower leg swelling that occurs after prolonged standing but resolves w/ sitting or lying down. Her last pregnancy was complicated by Group B strep colonization requiring prophylaxis during delivery. The pt has no sig PMH & takes only prenatal vitamins. She is a radiologist & does not use tobacco, drink, or use drugs. BP is 135/80, pulse is 65/min. PE shows minimal pedal edema & neg Homan sign bilaterally. Fundal ht is 27 cm and fetal HR is 140/min. Which of the following tests should be obtained at this visit? (Group B strep rectovaginal culture OR Oral glucose challenge test)

Oral Glucose Challenge test > should be administered at 24-28 wks Group B strep Culture > at 35-37 weeks

A 40F, G2P1, presents for her first prenatal visit. She is 10 wks, based on her last menstrual period. The pt has no medical problems & takes no meds. Her husband & 3 yrs old son are healthy, but she has a cousin w/ Down's. Vitals are normal. Urine B-hCG is positive & fetal heart tones are present. Which of the following is the most appropriate next step in MGMT? (order Cell-free DNA test OR Order AFP, Estriol, B-hCG, & inhibin-A Quad test)

Order cell free DNA test > pt is > 35 & is at increased risk of fetal aneuploidy & should be offered this test Order AFP, estriol, B-hCG, Inhibin-A test > If pt did not meet high-risk criteria (age <35) then could use this test

A 28F, being evaluated for infertility presents d/t to clear vaginal discharge for the past 2 days. The pt & her partner have intercourse EOD & have been trying to conceive for the past 7 months. Menstrual cycles occur every 29 days w/ 4 days of flow. Last week, She took penicillin for a sore throat. The pt has no medical issues & has had no surgeries. She takes a daily prenatal vitamin. Pelvic exam shows clear mucus at the cervical os. Which of the following is the most likely explanation for this pt's discharge? (Ovulation OR Cervical mucus plug OR Candidiasis)

Ovulation > Cervical mucus secretion close to ovulation increases in quantity & can be perceived by pts as vaginal discharge >> the mucus is clear, elastic, thin in consistency, & described similar to appearance to an uncooked egg Cervical mucus plug > would be brown, red, or yellowish thick mucus that is typically shed before or during labor Candidiasis > Causes a thick white to yellow discharge & mucosal erythema >> (not clear cervical discharge)

A 27F, G2P2, is evaluated on the postpartum floor d/t generalized tonic-clonic seizures that occurred 30 hrs after a vaginal delivery. The pt was induced at 38 wks via an oxytocin infusion for preeclampsia w/ severe features. She received a Mg2+ infusion for seizure prophylaxis that was discontinued 24 hrs after delivery. Her pregnancy was complicated by insulin-dependent DM & an insulin drip was used during labor. The pt had a spontaneous vaginal delivery complicated by a postpartum hemorrhage that was treated w/ bimanual massage & an oxytocin infusion & bolus. Temp is 98.7F, BP. is 128/82, Pulse is 78/min, RR are 18/min. PE reveals a lethargic pt w/ no focal deficits. Labs show: - Hb-------------10 - Plts------------160K - Na+------------112 - K+--------------3.6 - Cl--------------100 - Glucose-------78 - Ca+------------9.6 - Mg2+----------6.3 Which of the following is the most likely cause of the pt's seizures? (Mg2+ tox OR Oxytocin Tox)

Oxytocin > AE's include: 1- *Hyponatremia d/t water intoxication* >> structurally similar to ADH 2- Hypotension 3- Tachysystole Mg2+ Tox > Would present w/ hypo-reflexia, Lethargy, headache, resp failure, cardiac arrest

A 22F, presets to the ED w/ RUQ pain that has worsened over the past day. The pain initially began 5 days ago in the lower abdomen after she returned from a trip to South America. The RUQ pain increases w/ deep breathing. She now has fevers, chills, & vomiting. The pt has no diarrhea, constipation, or changes in stool color. She has no medical conditions or previous surgeries. The pt is sexually active w/ a male partner & does not use contraception. She is currently on her menstrual period but reports increasing irregularity over the last 3 months w/ occasional spotting. She does not smoke, drink, or use drugs. Temp is 102F, BP is 100/70, Pulse is 104/min. BMI is 21. Exam shows RUQ tenderness, & the lower abdomen is diffusely tender w/out guarding. Bowel sounds are present. There is no CVA tenderness. Skin is normal w/out a rash. Urine pregnancy test is negative. Which of the following is the most likely Dx? (PID OR Acute viral hepatitis OR Ruptured ovarian cyst)

PID > Typically Presents w/ 1- *Lower abdominal pain/tenderness* >> can spread to liver capsule & present as *RUQ pain* 2- Abnormal bleeding 3- *Fever, chills, vomiting* 4- Mucopurulent cervical discharge 5- Cervical motion & uterine tenderness > Intermenstrual spotting can occur d/t cervicitis >> abd pain d/t PID typically worsens w/ menses RF 1- *Age <25* Usually presents in young female 2- *Sexual activity w/out barrier protection* Acute viral hepatitis > Can present w/ RUQ pain >> But pts *typically have associated jaundice & changes in stool color* Ruptured ovarian cyst > Not associated w/ fever 1- Sudden onset *Abd pain w/Bleeding* (shock hemorrhage) 2- Nausea, vomiting 3- Dyspareunia

A 54F, G3P3m presents d/t difficulty voiding for the past few weeks. When the pt voids, she has difficulty initiating the urine stream & emptying completely, but has no dysuria or hematuria. The pts has 2 episodes of nocturia every night, but no involuntary leakage of urine. She also has pelvic pressure that is worse w/ standing. Ten months ago, the pt underwent a total abdominal hysterectomy w/ bilateral salpingo-oophrectomy for epithelial ovarian cancer & has completed a Course of chemo. Vitals are normal. BMI is 36. The abdomen is w/out masses or ascites. PE shows a protruding soft, non-tender mass at the level of the hymen that descends past the introitus w/ the valsalva maneuver, which does not cause leakage of urine. The uterus & ovaries are surgically absent. Sensation & deep tendon reflexes are intact. Urinalysis is normal. Which of the following is the most likely cause of this pt's symptoms? (Intrinsic sphincter deficiency OR Pelvic organ prolapse)

Pelvic organ prolapse > Symptoms include: 1- *Pelvic pressure* 2- *Voiding dysfunction* (*retention*, *difficulty imitating stream*, incontinence) 3- Bowel dysfunction PE findings > Vaginal mass that increases in size w/ valsalva Intrinsic sphincter deficiency > form of stress urinary incontinence >> causes *involuntary leakage & loss of urine w/ valsalva* > Pts have no urinary retention or any difficulty initiating micturition

A 46F, nulliparous, presents for a routine exam. She has been feeling well. Her last menstrual period was 1 week ago. The pt has regular menses every 30 days that last 3 days, w/ crampy abd pain on the first day relieved by ibuprofen. She has a history of heartburn, which is relieved by OTC antacids. The pt has never had any surgeries. She does not smoke, and drinks alcohol on social occasions. The pt is sexually active w/ her husband; they do not use contraception as he has had a vasectomy. Abd exam shows a symmetric, nontender, non distended abdomen w/ normal bowel sounds. Speculum exam shows a Cervix w/ a closed external os. Bimanual exam shows a 5 cm, right adnexal mass. Urine pregnancy test was negative. Which of the following is the best next step in MGMT? (Pelvic US OR Serum CA-125 level OR X-ray of abdomen & pelvis)

Pelvis US > US is the first line test for eval of a palpable adnexal mass XR pf abdomen & pelvis > not good for imaging pelvic organs b/c they are radiolucent & compromised of soft tissue CA-125 level > Once pelvic US confirms that the adnexal mass is ovarian >> then CA-*125 can be considered but mainly used in postmenopausal women and not recommended for the initial eval of an adnexal mass*

After a non-reactive stress test, a pt has a biophysical profile test and US confirms an anterior placenta covering the cervical os & an amniotic fluid index of 1.5 w/ a single fluid pocket measuring 1.5 x 1cm. Over a period of 45 mins there are 4 episodes of fetal mvmt, 3 flexion/ext events, and no fetal breathing mvmts. The fetus is breech. Which of the following is the most likely cause of the pts biophysical US findings? (Placental dysfunction OR Placental location)

Placental dysfunction > Pt has a decreased amniotic fluid index (should be single fluid pocket >/= 2cm x 1 cm or amniotic fluid index >5) >> pt has *oligohydraminos* > *Pts Biophysical profile score is 4/10* -- calculated by: (all 2 pts if normal) 1- NST---Normal 2- Amniotic fluid volume--Normal (above) 3 *>/=3 general body mvmts* 4 *>/=1 Flexion/ext of fetal limbs or spine* 5 >/= 1 breathing mvmt >/= 30 sec (no) * score of 4/10 signifies placental dysfunction* Placental location > Pts w/ anterior placenta more frequently report a decrease in fetal mvmts but usually b/c this position may make perception of mvmt more difficult >> *however there is no association btwn placental location & an abnormal BPP*

A 37F, brought to ER by BF for severe abd pain that started several hs ago. The pain started diffusely in the periumbilical area but is now mostly localized to the lower abd. The pt has persistent nausea but no vomiting. Her last menstrual period was 25 days ago, but she has irregular periods. She is sexually active w/ her BF. Temp is 98.1F, BP is 130/80, Pulse is 110/min, RR is 25/min. The abd, especially in the lower quadrants is tender on palpation w/ mild guarding. There is no shifting dullness. Bowel sounds are diminished. Labs show - Hb. 13.1 - Hct. 43% - WBC. 10,9 Which of the following is the most appropriate next step in MGMT? (Abdominal CT OR Pelvic US OR Pregnancy test)

Pregnancy test > should be given before any test that involves radiation (CT) Pelvic US > Pregnancy test first >> then pelvic US

A 38F, at 12wks presents with chronic HTN. The pt is at greatest risk of which of the following complications? (Preterm labor OR PPROM)

Preterm labor > Fetal Risk d/t HTN include: 1- FGR 2- Perinatal morality 3- *Preterm delivery* 4- Oligohydraminos PPROM > RF for PPROM are 1- Previous PPROM 2- Infection

A 55F, postmenopausal, presents for a routine health exam. She feels well. The pt was briefly hospitalized 6 months ago for bronchitis complicated by a rib fracture d/t severe coughing. She has no chronic medical conditions or previous surgeries. Her mother was Dx'd w/ osteopenia at age 70. The pt smokes 1/2 pack of ciggs daily until age 40 & eats a vegetarian diet. She drinks 2-3 cups of coffee every morning & a glass of wine each night. BP is 120/70, Pulse is 84/min, RR are 12/min. BMI is 30. PE is unremarkable. Which of the following is the most significant RF for bone fracture in the pt? (Osteopenia in mom OR Previous Rib fracture)

Previous Rib fracture > Pt's risk for fragility fracture (FRAX) has the highest risk in pts w/ *Prior Hx of fragility fracture* Osteopenia in mom > Hx of hip fracture in 1st degree relative is predictive of fracture risk >> However, Osteopenia is common in elderly women & *alone is not a significant familial RF*

A 23F, G1P0, at 41 wks, is admitted to L & D for contractions over the past 3 hours. The pt has no chronic Medical conditions. She has an elevated 1 hr glucose challenge test but a normal 3-hr glucose tolerance test at 28 wks. The pt receives epidural immediately upon admission & has SROM 2 hrs after admission. An intrauterine pressure catheter is placed for variable decels, which resolve w/ an amnioinfusion. Her cervical exam shows: - Dilation of cervix 8cm at 8hrs - Station +2 at 8 hours With no MVMT in either for the last 4 hours. Current temp is 98F. Fetal heart tracing shows a baseline of 140/min, moderate variability, & multiple accelerations. Intrauterine pressure catheter shows an average of 200 Montevideo units every 10 mins for the last 4 hrs. Which of the following is the best next step in MGMT? (Continue current MGMT OR Proceed to CS delivery)

Proceed to CS delivery > pt is in *active phase arrest* >> Criteria is: 1- *Cervix is >/= 4-10 cm (8cm) & is unchanged for the past 4 hrs despite adequate contractions* 2- No change in Cervix >/= 6hrs w/ inadequate contractions Continue curent MGMT > No b/c pt is in active phase arrest

A TVUS is performed to determine cervical length and assess preterm labor risk. It reveals a short cervix w/ a length of 1.8 cm. Which of the following is the best next step in TMT of this pt? (Corticosteroid stimulation of fetal pneumocyte development OR Progesterone mediated maintenance of uterine quiescence)

Progesterone mediated maintenance of uterine quiescence > During pregnancy, progesterone maintains uterine quiescence & protects the amniotic membranes against premature rupture >> *also decreases the rate of preterm labor in pts w/ short cervices or a history of preterm labor* Corticosteroids to stimulate pneumocyte development > Typically given at < 37 wks and given to pts who are at imminent risk for preterm labor *this pt has no signs of labor & is only 18 wks*

A 36F, nulliparous, presents to the ER after a syncopal episode whole at work as a surgical tech. Her colleagues gave her 1 L normal saline before she arrived. The pt is currently on day 10 of her menses, which have become progressively heavier & longer over the past year, at regular 28 day intervals. ROS is sig for frequent lightheadedness & palpitations. She is sexually active w/ her bf & takes Combined OCP's. Temp is 98F, BP is 100/60, Pulse is 112/min. PE shows an irregularly enlarged uterus 12 weeks in size. Hb is 4.9 & Mean corpuscular volume is 75fL. Urine pregnancy test is neg. Which of the following is the most likely cause of this pts anemia? (Endometrial hyperplasia w/ atypia OR Proliferation of smooth muscle cells w/in the myometrium)

Proliferation of smooth muscle cells w/in the myometrium > Abnormal uterine bleeding can be caused by *uterine fibroids* Endometrial hyperplasia w/ atypia > Typically present in post-menopausal women w/ *normal sized uterus*

A 20F, G2P2, is evaluated in the postpartum unit 8 hrs after vaginal delivery. She is experiencing fatigue, perineal discomfort, & bloody vaginal discharge. The pt has voided twice since delivery but finds it difficult to initiate urination. She presented w/ ROM at 40 wks &, after a prolonged induction, had an uncomplicated vaginal delivery under epidural w/ a 3rd degree laceration that was quickly repaired. The placenta delivered spontaneously after which the pt had an episode of rigors & chills for 30 mins that has not recurred. Estimated delivery-related blood loss was 500mL; the pt received an oxytocin infusion for 4 hrs after delivery. Temp is 100.2F, BP is 120/80, Pulse is 76/min. PE shows an intact perineal repair but bloody discharge & small blood clots on her perineal pad. Her uterus is firm, and the uterine fundus is at the umbilicus. Which of the following is the most appropriate next step in MGMT? (Administer antibiotics OR Provide routine postpartum care)

Provide routine postpartum care > Pt is presenting w/ normal findings postpartum >> including: 1- *Transient rigors/ chills* (shivering) 2- Peripheral edema 3- *Lochia rubra*(reddish brown vaginal discharge d/t normal shedding of the uterine decidua & blood) 4- Uterine contractions & involution *uterus becomes firm & globular w/ fundus typically 1-2 cm above or below the umbilicus* 5- Breast engorgement Administer Antibiotics > would be given if pt had postpartum fever d/t endometritis >> which presents w/ 1- Fever >> temp >/= 38C (100.4F) in first 24 hrs post delivery 2- Uterine tenderness 3- Foul-smelling lochia

A 35F, presents for her first prenatal visit. The pt has been married for 8 years & has tried to conceive for the past 8 yrs. For the past few wks, she has experienced morning sickness & has had abd distention & breast fullness. Her last menstrual period was 2 months ago & a home pregnancy test was positive. She has no medical problems & has had no surgeries. The pt takes a prenatal vitamin daily. She does not drink, smoke, or use drugs. BP is 110/60, Pulse is 72/min, BMI is 24. PE shows a soft, nontender, tympanic abdomen w/ no masses. Bedside US reveals a thin endometrial stripe. Two office pregnancy tests are negative. Which of the following is the most likely Dx? (Missed abortion OR Pseudocyesis)

Pseudocyesis > Pt presents w/ amenorrhea, breast fullness, morning sickness, & abd distention >> all symptoms of early pregnancy >> however, *negative urine pregnancy test and thin endometrial stripe exclude pregnancy* > Presentation is consistent w/ pseudocyesis RF include: 1- Hx of infertility 2- Prior pregnancy loss Pseudocyesis occurs when the somatization of stress affects the HPA axis & causes early pregnancy symptoms >> form of somatization & requires psychiatric eval Missed abortion > Pt would report a cessation of early pregnancy symptoms, also: 1- pregnancy test would be positive 2- US would reveal a nonviable intrauterine gestation

A 34F, presents for eval of 3 months of amenorrhea. She first began experiencing irregular periods a year ago, & now they have stopped completely. The pt has no headaches, visual changes, galactorrhea, hair loss, or fatigue. She has a Hx of non-hodgkin lymphoma for which she received chemo 5 years ago; there are no signs of recurrence. The pt currently takes no meds. FH is noncontributory. Her most recent Pap was a last year & all prior testing was normal. She has no Hx of STD's. The pt does not smoke, drink, of do drugs. BP is 120/80, pulse is 78/min. BMI is 28. PE reveals a non enlarged thyroid w/out masses. Cardiopulmonary exam is normal. Pelvic exam reveals a dry vaginal mucosa & a small, anteverted, mobile uterus w/ no adnexal masses. A pregnancy test is neg. Labs show: - FSH increased - LH. increased - Prolactin Normal - TSH. Normal What is this pts most likely Dx? Explain the lab findings in this pt?

Pt has Hx of cancer, amenorrhea, & signs of estrogen deficiency (vaginal dryness) >> findings are consistent w/ *Ovarian failure secondary to chemo* *Hypergonadotropic hypogonadism in women <40 (AKA primary ovarian insufficiency) occurs d/t absence of developing follicles >> *Chemo affects this* Labs - *Increased* FSH - *Increased* LH - Normal prolactin - Normal TSH

A 38F, G5P5, is evaluated fo difficult & painful ambulation on postpartum day 1. The pt has no pain while lying down but reports sharp lower mid-line abd pain that radiates down her legs while ambulating. She has voided twice w/out difficulty & has no dysuria. She has passed flatus but has had no BM's since delivery. She has not had numbness or foot drop. The pt received an epidural during labor & had a vaginal delivery 9lbs 8 oz that was complicated by shoulder dystocia relieved by the mcRoberts maneuver & suprapubic pressure. Temp is 99F, BP is 120/80, Pulse is 90/min, RR 18/min. Neurologic exam shows normal strength, sensation, reflexes of the LE. Abd exam shows focal tenderness to palpation just inferior to the bladder. Pelvic exam shows normal lochia, no vaginal masses, & a nontender uterine fundus that Is below the umbilicus. There is moderate edema bilaterally in the LE but no pain or erythema. Which of the following is the most likely Dx? (Pelvic abscess OR Pubic symphysis diastasis)

Pubic symphysis diastasis > During pregnancy, increased levels of progesterone & relaxin increase the pelvic mobility & promote a physiologic widening of the pubic symphysis >> after traumatic delivery, pts can develop a symptomatic pubic symphysis diastasis >> which presents w/: 1- *Suprapubic pain tat radiates to the back, hips, thighs, or legs* & is *exacerbated by walking*, weight bearing, or position changes 2- Pt's may have point tenderness to palpation over the pubic symphysis & sometimes have a waddling gait Pelvic abscess > Usually seen as a complication of CS >> presents w/: 1- Suprapubic/pelvic pain 2- *Fever* that is unresponsive to antibiotics *this pt had a vaginal delivery is afebrile*

The pt is admitted to the hospital, and an IV line & urinary catheter are placed. She receives corticosteroids to accelerate fetal lung maturity & mag sulfate for fetal neuroprotection. She develops dyspnea & a drop in arterial O2 sat 3 hs later. Temp is 98F, BP Is 150/80, Pulse is 112/min, RR is 24/min. The pts pulse ox shows 91% on room air. Exam shows bibaisilar crackles, use of accessory muscles for breathing, and 2+ pitting edema of the LE.; otherwise the exam is normal. Urine output is 60mL/hr. What is the most likely cause of this pts respiratory symptoms? (Pulm edema OR Mag sulfate tox)

Pulm edema > Pulm edema is a complication of preeclampsia >> presents w; - *Bibasilar crackles* - *Sudden onset dyspnea* - *Hypoxia* Mag sulfate tox >Will cause neuromuscular depression >> characterized by decreased resp effort/apnea, muscle paralysis, somnolence, visual disturbances, & decreased/absent deep-tendon reflexes (*pt has normal tendon reflexes & increased resp effort*)

A 29F, presents to the ER w/ chest pain & dyspnea for 10 days. The chest pain is bilateral, dull, persistent, & not worsened by exertion. The pt's PMHx is sig for a normal spontaneous vaginal delivery 3 months ago, after which she has had frequent episodes of dark, bloody vaginal discharge. The most recent bleeding episode was 6 days ago. The pt has no fever, chills, hemoptysis, orthopnea, or leg pain. She does not smoke, drink, or use drugs. Temp is 99.5F, BP is 110/68, pulse is 80/min, RR 16/min. BMI is 26.2. PE reveals bilaterally clear lungs. The extremities are warm & well perfused. There is no peripheral edema. Pelvic exam reveals an enlarged uterus, a closed cervix, & minimal dark blood in the vagina. CBC and serial electrolytes are w/in normal limits. CXR reveals multiple bilateral infiltrates of various shapes. Which of the following would be most helpful in establishing the pt's Dx? (Pulm function test OR CT scan of chest OR Echo OR Ventilation perfusion scan OR Quantitative B-hCG test)

Quantitative B-hCG test > Pt presents w/ *choriocarcinoma* >> Choriocarcinoma most commonly follows: 1- Hydatidiform mole 2- *Normal gestation* 3- Spontaneous abortion > typically presents *<6 months after a pregnancy* Symptoms include: 1- *Irregular vaginal bleeding* 2- *Enlarged uterus* 3- Pelvic pain 4- *Pulm Metastasis* (*chest pain*, hemoptysis, & *dyspnea*) Pulm function test/Echo/ Ventilation perfusion scan > would identify obstructive or restrictive disease >> but none would present w/irregular vaginal bleeding or enlarged uterus CT scan > Staging for choriocarcinoma relies on CXR not CT > would not do CT in a female of reproductive age if not needed Echo > nothing wrong with heart >> maybe if pt was older & had Hx of CHF

A 29F, G2P1, presents at 10 wks for her first prenatal visit. She feels well except for mild nausea. Her first pregnancy ended w/ an uncomplicated vaginal delivery, but she required penicillin for a vaginal infection during labor. The pt has no other med conditions & takes no meds. She does not drink, smoke, or use drugs. She has been in a monogamous relationship w/ her husband since they married 5 yrs ago. They live in a house built in 1983 & have 3 cats. Vitals are normal. Her PE is unremarkable. Which of the following screening is indicated for the pt at this visit? (RPR OR Toxo serum antibodies)

RPR > At first prenatal visit, pts should be screened for STD's, such as: 1- HIV 2- Hep B 3- Chlamydia 4- Syphilis Toxo serum antibodies > only tested for when pt has signs/ symptoms of toxo infection (Fever, malaise, lymphadenopathy)

A 24F, G1P1, at 37 wks presents requesting an induction of labor. The pt says that the last few weeks of pregnancy have been "very uncomfortable" d/t exhaustion & SOB when walking. She has bilateral leg swelling & is unable to fit into her shoes. The pt has difficulty sleeping at night d/t bilateral crampy pain in her calves that is relieved by massage & stretching, & b/c the baby moves frequently at night. ROS is negative for contractions, vaginal bleeding, & loss of fluid. Temp is 36.8c (98.2F), BP is 110/70, Pulse is 110/min, RR is 18/min. Urinalysis shows trace protein. PE shows symmetrical, bilateral pitting edema of the calves, ankles, & feet. Cardiac auscultation shows a 2/6 systolic ejection murmur. Her lungs are clear to auscultation. Which of the following is the most appropriate next step? (24 urine collection for total protein OR reassurance & follow-up)

Reassurance and follow up > Pregnancy causes physiologic changes such as: 1- Faster HR >> frequently accompanied by *Systolic ejection murmur* 2- Peripheral edema >> d/t increased plasma volume 3- Dyspnea d/t reparatory changes (decrease in FRC) 4- Nocturnal leg pain (d/t muscle cramping from lactic & pyrite acid accumulation) > TMT *Reassurance & routine prenatal care* 24 urine collection for total protein > Part of eval for preeclampsia >> which would present w/: 1- HTN 2- Edema 3- Pathologic proteinuria

A 41F, G2P1, at 35 wks gestation presents to the ED w/ uterine contractions that started approximately 6 hrs ago after a trip to the beach. The patient feels 3-5 contractions every hour. She has been managed w/ weekly injections of hydroxyprogesterone for a history of vaginal delivery after presenting in spontaneous preterm labor at 34 wks gestation. Two days ago, the pt had a routine rectovaginal culture that was positive for GBS. BP is 110/80, pulse is 92/min. Digital cervical exam shows the cervix to be closed. Bedside US shows an estimated fetal weight consistent w/ 35 wks gestations & a vertex presentation. The fetal heart tracing is normal & tocometry shows irregular uterine contractions. Which of the following is the most appropriate next step in MGMT of this pt? (Administer indomethacin OR Admninister betamethasone OR Reassure the pt and discharge)

Reassure & discharge > Pt presents at 35 wks w/ *irregular contractions & a closed cervix* consistent w/ false labor Betamethasone > given in pts with preterm delivery <37 wks >> but pt is not likely to be in labor Indomethacin > Not given after 32 wks >> *Tocolysis not indicated in pts after 34 wks*

A 53F, presents for a routine exam. Her last menstrual period was 2 yrs ago. The pt has hot flashes throughout the day & awakens several times each night drenched w/ sweat. Wt loss & OTC meds have not improved her symptoms. PMHx include HTN & DM II controlled w/ diet & exercise. The pt's mom had a bilateral hip replacement after fall at age 87, & her father had Az disease. BP is 124/80, Pulse is 76/min, BMI is 28. MSE is normal. Cardiopulmonary exam is normal. PE shows a small uterus and nontender, & there are no adnexal Masses. The extremities has 2+ peripheral pulses. HbA1c is 6.9%. Which of the following is an indication for systemic hormone replacement therapy in this pt? (Reduction of vasomotor symptoms OR No indication)

Reduction of vasomotor symptoms > Pt has *severe hot flashes - vasomotor symptoms* (compared to mild hot flashes which do not interfere w/ daily activities) > Severe hot flashes (pt wakes up drenched in sweat) are the only indication for HRT in women <60 who have undergone menopause in the last 10 years

A 20F, presents to the ER d/t to vaginal bleeding & RLQ pain that began 3 days ago. She describes the bleeding as heavier than a period, and she passed vaginal clots 3 hrs prior to presentation. Her menarche was at age 13 & she often goes months w/out a menstrual period. The pt's last period began approximately 7 wks ago. She is sexually active & uses the withdrawal method for contraception. Temp is 98.9F, BP is 120/74, Pulse is 80/min. Exam shows mild RLQ tenderness, but no rebound or guarding. There is no active vaginal bleeding & the cervical os is closed. Labs show: - Hb-----------------11g/dL - B-hCG-------------1000 - Blood type--------AB - Rh factor----------Positive A transvaginal US reveals no intrauterine or extrauterine pregnancy. Which of the following is the best next step in MGMT of this pt? (Perform abd US OR Repeat serum B-hCG in 2 days)

Repeat serum B-hCG in 2 days > Pt positive pregnancy test but no evidence of an intrauterine or extrauterine pregnancy >> differentials include: 1- Early viable intrauterine pregnancy ectopic pregnancy 2- Non-viable intrauterine pregnancy (competed abortion) > *Serial B-hCG levels should be ordered when the transvaginal US is indeterminate* Perform abd US > Much less sensitive in detecting an early pregnancy & would not be useful if a TVUS did not reveal a gestational sac

A 39F, G1P0, at 32wks presents for routine prenatal care. She has had fetal mvmts but no contractions, abd pain, headaches, or changes in vision. The pt has HTN Dx'd at 26 wks that is being managed w/ labetalol & weekly BPP's. An eval for preeclampsia was negative at the time her HTN was Dx'd. Current BP is 135/85 & pulse is 72/min. A urinalysis shows trace protein. The NST test shows moderate variability, accelerations, and no decels. BPP score is 10/10. Which of the following is the most appropriate next step in mgmt? (Repeat testing in 1 week OR Repeat testing in 24hs)

Repeat testing in 1 week > Pt would only need to repeat testing in 24 hours in BPP score was 6/10>> this pt was a BPP of 10/10

In previous pt (that has uterine inversion and hemorrhagic shock), Two wide bore IV lines are placed. Crystalloid fluids & blood products are administered. Which of the following is the best next step in MGMT? (Admin of uterotonics OR Replacement of the uterus)

Replacement of uterus > MGMT includes: 1- Aggressive fluid replacement 2- *Manual replacement of uterus* 3- Placental removal & unterotinic drugs after uterine replacement Admin of uterotonics > replace uterus first then admin uterotonics

A 42F, presents d/t recurrent abd bloating, fatigue, & hot flashes that improve after menses. These symptoms have been occurring for the past 4 days & make her feel "extremely cranky". The pt has missed work d/t these symptoms. Her last menstrual period was 3 wks ago. Menses occurs every 30 days for 6 days, w/ 2 days of heavy flow. She has a PMHx of migraines w/ aura for which she takes a beta blocker. The pt takes no other meds & has NKA. She is monogamous w/ her bf & uses spermicidal foam for birth control. PE is normal. Which of the following is the most appropriate recommendation for this pt? (Combined oral contraceptives can improve these symptoms OR Return to the office in 2 months w/ symptom diary)

Return to the office in 2 months w/ a symptom diary > Pt has PMS >> a symptom over 2 menstrual cycles should demonstrate recurrence of symptoms during the luteal phase (1-2 wks prior to menses) & resolution during the follicular phase (onset of menses or a few days thereafter) Combined oral contraceptives can improve these symptoms > Can work >> *but estrogen containing meds are CI in pts w/ aura*

A 26F, Primigravid, at 20 wks presents to the ER d/t joint pain & swelling in her hands & feet over the past day. The pt reports good fetal mvmt & has no abd pain, bleeding, leakage of fluid, headaches, or visual changes. She has a Hx of SLE that was managed w/ hydroxychloroquine, which she stopped taking prior to conception. BP is 150/95; 5 mins later, following rest in the lateral recumbent position, it is 140/95. Fetal HR by doppler US is 145/min. PE shows bilateral 2+ pitting edema of the legs & a red macular eruption of the cheeks. Urinalysis shows - Urine protein------2+, RBC casts - Urine protein-------1.5g - LDH-----------------62U/L Serum electrolytes, liver function tests, & coagulation studies are normal. Which of the following is the most likely Dx? (Preeclampsia OR SLE flare)

SLE flare > Pt at 20 wks has: 1- Edema 2- Joint pain 3- *Malar rash* 4- HTN 5- urinalysis with *RBC casts* & proteinuria >> *All consistent w/ SLE nephritis* > >Also has RF b/c she discontinued Hydroxychloroquine meds >> Sig Hx for SLE Preeclampsia > Will also presents w/ Edema, HTN, Proteinuria >> *but not malar rash, joint pain, RBC casts on urinalysis*

A 34F, G2P2 at 34 wks, presents to establish prenatal care. The pt recently moved but had routine prenatal care in another state. She has had no contractions, vaginal bleeding, or leakage of fluid. Fetal mvmt is normal. The pt had a classical CS at 25 wks d/t eclampsia & FGR. This pregnancy has been uncomplicated, & her only meds are a daily aspirin & a prenatal vitamin. BP is 118/70. Fetal Heart tones are 145/min & fundal Ht is 38 cm. Urine dipstick is neg for protein & glucose. An US is performed d/t size/date discrepancy. The US shows a posterior placenta, normal amniotic fluid volume, & a fetus in frank breech presentation w/ an estimated fetal wt at the 50th percentile for gestational age. The pt would like to attempt a vaginal delivery w/ this pregnancy. Which of the following is the most best next step in MGMT of this pt? (Perform EVC at 36 wks OR Schedule a CS at 37 weeks)

Schedule a CS at 37 wks > In pts w/ *prior classical CS delivery* a vaginal delivery is contraindicated d/e to increased risk of uterine rupture >> in this pts an ECV is not performed & they require a CS delivery

A 26F, presents for an infertility eval w/ her husband, age 30. They have been unable to conceive after 12 months of unprotected intercourse. The pts' periods are regular, occurring every 26 days w/ painful cramps the first days that are relieved w/ ibuprofen. Her last menstrual period was 6 days ago. She had 3 previous sexual partners w/out pregnancy. Both the pt and her husband report normal puberty, & neither have had any medical conditions or previous surgeries. The pt takes a prenatal vitamin. She is 5ft 1in tall & weighs 121.3 lbs. BMI is 28, BP is 120/70, pulse is 70/min. PE shows Tanner stage 5 breast development. PE shows a normal cervix; mobile, retroverted, nontender uterus; & no adnexal masses. PE of the husband shows normal secondary sexual characteristics & bilaterally descended testes. Urine pregnancy test is negative. Which of the following is the best next step in eval of the infertility? (Hysterosalpingography OR Semen analysis)

Semen analysis > gold standard for mens eval for infertility >> up to 25% of cases of due to male >> *pt has no signs or RF's for any abnormalities* Hysterosalpingography > Not first line for eval of infertility >> would only be used if pt has RF's for tubal blockage *PID, endometriosis*

A 34F, G2P1, presents to the L & D unit at 38 wks w/ contractions & spontaneous rupture of membranes w/ green fluid. The pt lives in a homeless shelter. Her only medical care during they pregnancy was an ER visit for spotting at 11 wks. An US at that time confirmed her due date. Her first pregnancy, 6 yrs ago, was complicated by GDM & ended w/ a vaginal delivery of a 4kg (8.8 lbs) infant. The pt had an elevated 2-hr glucose tolerance test 6 wks after that delivery but did not follow up afterwards. Today BP is 150/90. Finger stick glucose is 179mg/dL. The fetal HR tracing shows minimal variability but no decels. The pt delivers a 2.2kg (4.8 lbs) girl w/ Apgar scores of 6 & 9 at 1 & 5 mins. On PE, the infant appears thin w/ loose skin & a large anterior fontanel. No retractions or grunting is present. The placenta is meconium stained & the umbilical cord is thin. In addition to routine neonatal care, which of the following is the best next step in MGMT of this infant? (Perform cranial US OR Send placenta for histopathologic exam OR Start infant on antibiotic therapy)

Send placenta for histopathologic exam > Infant has signs of fetal growth restriction, which present as: 1- *Thin, loose skin* 2- *Thin umbilical cord* 3- *Wide anterior fontanel* 4- *Meconium-stained amniotic fluid* 5- Decreased sub-q fat > Placenta should undergo histopathologic exam to evaluate for the presence of infarction and/or infection (spirochetes) Perform cranial US > Used to Evaluate for intraventricular hemorrhage >> indications include a gestational age < 30wks (pt is 38 wks) 2- Resp distress (Tachypnea, grunting) 3- Hypotension *This infant has a normal 5 mins Apgar score w/ no evidence of resp. distress* Start on antibiotics therapy > Indicated for TMT of neonatal sepsis >> meconium stained amniotic fluid is RF >> however, neonatal sepsis typically presents w/: 1- Fever 2- Tachypnea 3- Grunting 4- Lethargy

A 29F, G1P1, is evaluated d/t recurrent fevers on day 5 after CS for arrest of descent after a prolonged induction of labor. Gentamicin & Clindamycin were administered for fever on post-op day 1. Ampicillin was added on post-op day 3 as the pt continued to be intermittently febrile. Today, she has no nausea, vomiting, hemoptysis, dyspnea, hematuria, dysuria, or diarrhea. Temp is 102.2F, BP is 120/80, Pulse is 108/min. O2 sat is 96% on room air. The lungs are clear to auscultation bilaterally. Breast exam shows full, slightly tender breasts w/ no erythema. Abd exam shows mild bilateral LQ tenderness to deep palpation & an incision w/ serosanguineous drainage but no erythema or induration. PE shows a nontender uterine fundus that is below the umbilicus. Exam of the lower extremities shows no swelling, tenderness, or erythema. Hb is 10.8 g/dL. Urinalysis is normal. CT of the abd & pelvis shows no abscess or fluid collection. Which of the following is the most likely Dx? (Postpartum endometritis OR Septic thrombophlebitis)

Septic thrombophlebitis >Dx of exclusion >> Presents w/: 1- *fever unresponsive to antibiotics* 2- Negative infectious values *bloods, urine cultures* 3- No localizing signs/symptoms RF include 1- *CS delivery* 2- Pelvic surgery 3- Endometritis 4- PID 5- Pregnancy 6- Malignancy Postpartum endometritis > Most common cause of post delivery fever >> But would also have 1- Purulent lochia 2- Uterine tenderness 3- Would respond to antibiotic therapy

A 22F, nulligravid, presents for worsening acne on her face and body over the last 6 months. She initially attributed the acne to her diet & has been eating healthier to improve the acne & lose weight. The pt has lost 15lb in the last 3 months, but the acne has not improved. She has no chronic medical conditions & has had no surgeries. The pt does not use tobacco, alcohol, or drugs. Her father has DM II, & mom has hyperlipidemia. BP is 138/86, pulse is 72/min, BMI is 42. Coarse hair is noted on the chin & upper lip. Nodulocystic acne is present on the face, chest. & back. Pelvic exam shows an enlarged clitoris but otherwise normal external genitalia. Bimanual exam is limited by body habitus. Labs show elevated total testosterone & normal dehydroepiandrosterone. Which of the following is the most likely Dx? (PCOS OR Sertoli leydig cell tumor)

Sertoli leydig cell tumor > Pt has *severe hyperandrogenism* >> presents w/: 1- *Markedly elevated testosterone levels* >> defined by either: ----- *Virilization* OR -----*Rapid onset hyperandrogenesim* >> Changes Occurred in Pt in *6 months* >> suggests an androgen-secreting tumor PCOS > would not show elevated testosterone levels that result in virilization >> would have *MILDLY elevated testosterone levels*

A 62F, Presents for a follow-up visit right after adnexal enlargement was found on routine pelvic exam 2 wks ago. Pelvic US reveals a 5-cm right ovarian cyst. Menopause occurred at age 52 & the pt has had no postmenopausal spotting or bleeding. She had an abnormal Pap in her 20's that subsequently tested as normal & her PMHx is unremarkable. The pt has never had any surgeries. There is no FHx of ovarian or breast cancer. Her most recent mammogram 2 months ago showed no abnormalities. Which of the following is the most appropriate initial course of action for this pt? (Endometrial biopsy OR Serum CA-125 level)

Serum CA-125 > Pt presents w/ ovarian cancer >> used to categorize an ovarian mass as likely malignant or benign Endometrial biopsy > Would be indicated if pt had 1- Postmenopausal bleeding 2- Abnormal uterine bleeding 3- Thickened endometrial stripe + ovarian mass

A 29F, Nulligravid, Presents b/c she has not menstruated since stopping her birth control. Menarche was at age 14, & she had irregular, heavy menstrual periods from age 14-16. The pt then took combo oral contraceptives until 6 months ago. Since stopping her meds, she has had more frequent headaches & increasing acne but has had no wt changes, pelvic pain, or abnormal hair loss or growth. The pt is a high school teacher & coaches the cross-country & track teams. She runs recreationally & eats a well-balanced diet. She does not smoke, drink, or use drugs. Vitals are normal w/ BMI 22. A few open comedones are seen on the forehead. The remainder of the PE, including pelvic exam is normal. Urine pregnancy test is neg & pelvis US is normal. Which of the following is the best next step in evaluation of this pt? (Serum 17-hydroxyprogesterone OR Serum prolactin levels)

Serum prolactin levels > This pt has secondary amenorrhea >> most common cause is pregnancy, followed by HPO axis dysfunction & endocrine abnormalities >> therefore the next best tests must eliminate theses causes, the tests are: 1- FSH 2- TSH 3- Prolactin 4- Pituitary/thyroid functions *acne* is seen w/ hormones and would not mean its d/t increased testosterone levels >> *cessation of Contraceptives typically worsens acne* Serum 17-Hydroxyprogesterone > Elevated in Non-classic CAH (21) which would cause *abnormal uterine bleeding & hyperandrogenism* > Would have electrolyte abnormalities (HTN) > Would have symptoms since birth w/ virilization

A 21F, presents to the ER d/t continuous vomiting, diarrhea, & high fever for a day. The pt also has a severe headache unrelieved by acetaminophen; she has no hematemesis or hematochezia. She returned yesterday on an international flight after spending the last 3 months in Southeast Asia. She has no chronic medical conditions or previous surgeries. The pt is sexually active & uses oral contraceptive. She is currently on the third day of her menstrual period. The pt does not use tobacco or illicit drugs. Temp is 103F, BP is 80/40, Pulse is 124/min. PE shows a diaphoretic woman w/ dry mucous membranes. There is no nuchal rigidity. An erythematous macular rash covers the entire body, including the palms & soles. Which of the following is the most likely microorganism responsible for these symptoms? (shigella OR Staph aureus)

Staph Aureus > pt presents w/ toxic shock syndrome, clinical features are: 1- Fever > 102F 2- *Hypotension* 3- *Diffuse macular rash involving the palms & soles* 4- Desquamation 1-3 wks after disease onset 5- Vomiting, diarrhea 6- Altered mentation w/out focal neurological signs Shigella > pt would have fever, bloody diarrhea, & abd cramping >> *No associated rash*

A 29F, G2P2, at 10 wks presents to the ED for vaginal bleeding w/ passage of large clots & intense lower abd cramping. The bleeding began 3 hrs ago & she reports worsening dizziness on standing. The pt initiated prenatal care a week ago & an US at that Time revealed a 9-wks gestation w/ a normal fetal heart rate. Temp is 98.7F, BP is 90/65, Pulse is 110/min, RR 17/min. Large clots of blood are evacuated from the vagina during pelvic exam, after which active bleeding from an open cervical os is noted. Blood type is AB neg. Hb is 8.0g/dL. Bedside TVUS reveals a 9-wks sized fetus in the lower uterine segment w/ no cardiac activity. IV fluids are begun. Which of the following is the most appropriate next step in MGMT? (Expectant MGMT OR Suction curettage)

Suction Curettage > Pt presents w/ *inevitable* abortion, presents w/: 1- Heavy vaginal bleeding 2- Cramping 3- Dilated cervix 4- US that shows no heartbeat 5- Intrauterine gestation in the lower uterine segment MGMT of inevitable abortion is dependent on patient preference & hemodynamic stability > Hemodynamically unstable pts *Like this pt* b/c (BP 90/60, Tachycardia, anemia from acute blood loss) >> *Suction curettage is indicated* Expectant MGMT > Appropriate in hemodynamically stable pts w/ minimal bleeding

A 32F, presents to ED w/ abd pain & nausea that began 2 days earlier but has become increasingly severe over the last 3 days. The pt has passed several blood clots vaginally for the last hour. She has a Hx of irregular menstrual cycles & is not sure of the date of her last period. She was Dx'd w/ "heart-shaped uterus" 2 years ago. BMI is 28. BP is 90/55, pulse is 120/min. Abd exam shows guarding w/ decreased bowel sounds. Speculum exam shows moderate vaginal bleeding w/ clots. A urine pregnancy test is positive. TVUS shows a gestational sac at the upper left uterine Cornu & free fluid in the posterior cup-de-sac. Which of the following is the most appropriate next step in mgmt? (Dilation & curettage OR Surgical exploration)

Surgical exploration > Pt presents w/acute abdomen (guarding w/ decreased bowel sounds) >> likely d/t rupture ectopic pregnancy *known as interstitial ectopic pregnancy* > The presence of fluid in the posterior-cup-de-sac in the setting of ectopic pregnancy suggests blood in the pelvis>> *surgical exploration is required in a pt with hemoperitoneum & unstable vitals* Dilation & curettage > Spontaneous abortion does not presents w/ acute abdomen >> this pt more likely suffered from an ectopic pregnancy in the left uterine cornual

A 42F, G2P2, presents for eval of pelvic pain. The pt's menses are painful w/ heavy bleeding that requires her to change her tampon every hours during the first 2 days. She did not have painful periods until a few years ago. The pelvic pain used to subside after menses but has become constant over the past few months & is unrelieved by ibuprofen. Her cycles are regular, occur every 28-30 days & last 4 days. She has had no dysuria, urinary frequency, or constipation. The pt had a tubal ligation after her last delivery. She has no had cervical cancer screening w/in the past 5 years & is not sexually active. Temp is 98F, BP is 120/70, pulse is 78/min. Exam shows a boggy uterus that is tender to palpation. Which of the following additional findings is most likely present in this pt? (Posterior cul-de-sac nodularity OR Symmetrically enlarged uterus OR Large, irregularly shaped uterus)

Symmetrically enlarged uterus > pt presents w/ chronic pelvic pain & heavy menstrual periods likely d/t *Adenomyosis* >> which typically presents in *Women > 40yrs* with symptoms of: 1- *Symmetrically enlarged uterus* 2- New-Onset dysmenorrhea 3- Heavy menstrual bleeding 4- Chronic pelvic pain 5- Boggy, tender uterus Large, Irregularly shaped uterus > Common w/ fibroids (uterine leiomyomas) >> Cause heavy menstrual bleeding but *Uterus is firm*, not boggy Posterior cul-de-sac nodularity > Seen in pts w/ endometriosis >> can present with all symptoms seen in this pt >> however, Endometriosis is seen outside uterus, so would *not cause enlarged/boggy uterus*

A 29F, G1P0, presents at 16 wks to establish prenatal care. She has a history of cold knife conization 2 yrs ago for high-grade CIN. Other than obesity, she has no other med problems. The pt smoked but quit once she learned she was pregnant. She does not drink or use drugs. Temp is 98.6F, BP is 100/64, Pulse is 72/min. The fetal heart rate is 140/min. Which of the following is the best next step in evaluating this pt's risk for preterm labor? (Transabdominal US OR TVUS)

TVUS > The strongest RF for preterm labor is a prior preterm labor >> *first step in determining risk or preterm labor is TVUS measurement* of the cervical length in the second trimester >> *short cervical length is a strong predictor of preterm labor* Transabdominal US > Can measure cervical length but it is less accurate than TVUS d/t (maternal obesity, shadowing from the fetus, variations in cervical position)

A 32F, primigravid, at 18 wks is evaluated in the ER for increasing confusion & incoherence. Her husband says that she has become increasingly unsteady while standing & has fallen twice. The pt was previously seen in the ED for nausea & vomiting & was treated w/ IV fluids & antiemetics. Although she continues to take oral antiemetics, she has had persistent vomiting & has lost 7 kg (15.4 lbs) of her pregnancy wt. BP is 110/60 & pulse is 98/min. Fetal heart tones are 155/min. PE shows nystagmus but no scleral icterus. Pupils are reactive & equal to light & accommodation. Abd exam shows epigastric pain but no rebound or involuntary guarding. The pt has trace pedal edema over her bilateral LE & bilaterally absent ankle reflexes. Labs show: - MCV----------------84 - Na+-----------------131 - K+-------------------3.2 - HCO3--------------36 - AST-----------------110 - ALT-----------------114 - Lipase---------------32 Which of the following is the most likely Dx? (Thiamine deficiency OR Vit. B12 deficiency)

Thiamine deficiency - Pt presents Wernicke's encephalopathy >> presents w/ 1- Altered mental status (encephalopathy) 2- Oculomotor dysfunction (nystagmus) 3- Gait ataxia Associated conditions 1- Chronic alcoholism 2- Malnutrition 3- *Hyperemesis gravidum* Vit. B 12 deficiency > Presents w/ dementia and a subacute combined degeneration d/t demyelination of spinocerebellar tracts (gait ataxia) >> *However, clinical course is indolent rather than acute* > Also *Macrocytic anemia would be present* (pt has MVC <100)

A 27F, G1P0, at 28 wks presents b/c she has not felt fetal mvmts for the past 2 days. Aside from a Hx of obesity, she has no other sig PMH & her pregnancy has been uncomplicated. The pt's blood type is O, Rh-. Prenatal US at 12 wks showed an intrauterine gestation consistent w/ dates. An US fetal anatomical survey at 19 wks showed no abnormalities. Vitals are normal. Fundal ht is 24cm. Fetal heart tones are not heard by doppler. Which of the following is the most appropriate next step in mgmt? (NST OR Transabdominal US)

Transabdominal US > Pts typically present w/ decreased or absent fetal mvmt w/ RF including: 1- *Nulliparity* 2- *Obesity* 3- HTN 4- DM NST > uses the same technology as doppler >> so doing this would be the same thing as doing the doppler again

A 6-hr-old boy presents in the newborn nursery w/ tachy. He was born at 40 wks to a 30F w/ Graves' that was treated w/ surgical resection approximately 6 months before the pregnancy. Subsequently, the mom became mildly hypothyroid & was treated w/ levothyroxine during pregnancy. The infant's wt is 2080g (4lbs 9oz -- <5th percentile). Temp is 99F, pulse is 190/min. Exam shows an irritable but consolable infant w/ warm, flushed skin. Which of the following is the most likely cause of this infant condition? (Transplacental levothyroxine OR Transplacental TSH receptor Antibody)

Transplacental TSH receptor antibody > Pt presents w/ neonatal thyrotoxicosis d/t antibodies of Graves' crossing the placenta >> which presents as: 1- *Warm, moist skin* 2- *Tachy* 3- *Poor feeding, irritability, poor wt gain* 4- Low birth weight or preterm birth Transplacental levothyroxine > *Does not cross placenta* >> fetus Makes own TH

A 1-hr-old boy, is evaluated in the newborn nursery for microcephaly. He was born via spontaneous vaginal delivery to a 26F, who did not receive prenatal care. The mother has no known chronic med conditions. On PE, the boy's occipitofrontal circumference is >3 Std deviations below the mean. The anterior fontanelle is closed, & the skull is partially collapsed. Cardiopulmonary exam is normal. The abd is soft & nontender and has no palpable Masses. Multiple contractors & a right club foot are noted. The infant has marked hypertonia. Imaging shows a thin cerebral cortex. Which of the following maternal RF's is most commonly associated w/ the infant's presentation? (Consumption of unpasteurized dairy products OR Travel to a tropical, mosquito-infested region)

Travel to a tropical, mosquito infested region > Pt has Zika, which presents w/ clinical features such as: 1- Microcephaly, craniofacial disproportion 2- Neurological abnormalities (spasticity, seizures) 3- Ocular abnormalities > Imaging >> *Calcifications*, ventriculomegaly, *cortical thinning* Consumption of unpasteurized dairy products > pt would has Listeria >> in utero, pts would present w/: 1- Fetal demise 2- Neonatal disseminated granulomatous disease If congenital >> can cause intracranial calcifications but pts typically *have hydrocephalus*

A 40F, presents to the ED d/t abdominal pain for the past 2 wks. The pain has become worse over the past day & is not relieved by ibuprofen. The pt has a fever but no nausea, vomiting, diarrhea, of dysuria. She has no chronic medical conditions or previous surgeries. Her last menstrual period was 2 wks ago; menses are regular & last 5 days, w/ painful cramping & heavy bleeding on the first day. The pt is currently sexually active w/ a male partner & uses condoms for contraception. She does not smoke, drink, or do drugs. Temp is 102F, BP is 100/60, Pulse is 92/min. The abd has diffuse tenderness to deep palpation that is worse in the lower right quadrant. There is no rebound or guarding. PE reveals a small, mobile tender uterus & an enlarged, tender right adnexal mass. Leukocyte count is 22K. C-reactive protein & CA-125 are elevated. Urine pregnancy test is negative. Pelvis US reveals a large thick-walled multiloculated mass filled w/ debris obliterating the right adnexa. Which of the following is the most likely Dx? (Ovarian torsion OR Tubo ovarian abscess OR Ovarian cystadenocarcinoma)

Tubo ovarian abscess > Complications of PID >>Typically d/t a polymicrobial infection >> presents w/: 1- *Fever* 2- Abd pain 3- *Complex multiloculated adnexal mass w/ thick walls and internal debris* > Labs are non-specific & include: 1- Leukocytosis 2- Increased C-reactive protein 3- Increased CA-125 Ovarian torsion > presents w/ sudden onset unilateral pelvic pain *w/out fever or leukocytosis* > US findings would show an adnexal mass w/ absent doppler flow to ovary Ovarian cystadenocarcinoma > Would have same labs but *no fever* > would be described as necrotic w/ glands or higher gland ratio

A 41F, presents d/t loss of urine w/ coughing, sneezing for 4 months. She started wearing a sanitary pad a few days ago d/t urine leakage occurring a few times daily. The pt has seasonal allergies & has had frequent coughing & sneezing since the start of the spring season. She takes fexofenadine daily & no other meds. The pt is sexually active w/ a new partner & uses condoms. Menses are regular, & her last menstrual period was 2 wks ago. At age 25, she was treated for chlamydia. She has smoked 10 ciggs a day since her teens. Her mom has DM II. The pt's BP is 110/70, Wt is 121 lbs, Ht is 5ft 4 in, BMI is 21. PE shows an irregularly enlarged anteverted & ante flexed uterus. Which of the following would most likely reveal the cause of the pt's urinary symptoms? (US of pelvis OR Cystoscopy)

US of pelvis > Pt has stress incontinence >> and given her enlarged uterus w/ a irregular contour, most likely d/t leiomyomata uteri (fibroids) > *Dx of fibroids is made via US* Cystoscopy > Used to visualize the bladder directly for cancer

A 33F, presents for eval of a vaginal mass. The pt first noticed the mass 2 wks ago after having pain during intercourse. She has avoided sex since then, but the mass is still present. The pt has had no irregular menstrual bleeding or post coital bleeding. She has had 4 sexual partners in the last year & uses combo oral contraceptives. The pt had 3 term vaginal deliveries in her 20's; her largest baby was 10lbs. She had an abnormal cervical cancer screening 2 years ago but a normal follow-up colposcopy. Temp is 98F, BP is 120/70, Pulse is 76/min. BMI is 26. On pelvic exam, there is a 3-cm mass on the anterior aspect of the vaginal wall that does not change in size w/ Valsalva maneuver. The mass is tender to palpation & expresses a purulent discharge. Which of the following is the most likely Dx? (Bartholin gland abscess OR Urethral diverticulum)

Urethral diverticulum > The pts anterior vaginal wall mass is d/t urethral diverticulum (an abnormal out pouching if the urethral mucosa into surrounding tissues >> tender vaginal wall mass may present as dyspareunia or a palpable mass on pelvic exam Bartholin gland abscess > Located at the posterior vulvar vestibule >> not along the anterior vaginal wall

A 49F, G5P5, presents d/t involuntary intermittent loss of urine over the past 5 months. The pt is an avid jogger & now must wear absorbent pads while jogging. She has no dysuria, urgency, or hematuria & usually awakens once a night to void. 3 years ago, the pt was Dx'd w/ DM II, which is treated w/ metformin. She does not drink, smoke, or use drugs. Vitals are normal. BMI is 31. Pelvic exam shows normal external genitalia, a well-rugated vagina & an anterior vaginal bulge. The pt loses a small amount of urine when asked to cough. Post-void residual urine volume is 40mL. Clean catch urinalysis results show are normal. Serum HbA1c is 7.7%. Which of the following is the most likely cause of this pt's symptoms? (Bladder outlet obstruction OR Detrusor overactivity OR Urethral hypermobility)

Urethral hypermobility > Pt has stress urinary incontinence >> presents w/: 1- Leakage w/ valsalva (*coughing*, sneezing, laughing) Physio >> Substantial weakness of the pelvic floor muscles can result in urethral hypermobility >> in which the urethra abnormally moves w/ *increased intra-abdominal pressure* such as when *jogging*, *coughing* >> in addition, inadequate bladder support can develop prolapse *pt has anterior vaginal bulge - cystocele* Post-void volume > would be normal *<150 mL women* & <50mL in men Bladder outlet obstruction > Can be d/t diabetic nephropathy >> but would cause *overflow incontinence >> constant dribbling* & markedly elevated post-void volumes Detrusor overactivity > Causes inappropriate bladder spasms associated w/ *urge incontinence* >> pt would have a sudden need to urinate followed by immediate, involuntary loss of urine

A 31F, G1P0, at 8wks presents in ER d/t persistent nausea, vomiting, epigastric pain, & dizziness. The pt has been unable to tolerate any oral intake for the past day. This pregnancy was attained through Intrauterine insemination w/ donor sperm. She has a history of esophageal reflux for which she takes an OTC antacid. Prior to pregnancy, the pt drank socially but stopped alcohol use before her insemination. Temp is 98F, BP is 90/50, Pulse is 114/min. Current wt is 121.3 lbs, a decrease of 3.6 lbs from her pre-pregnancy wt. PE shows dry mucous membranes & decreased skin tumor. Cardiac exam shows tachy but a regular rhythm w/ no murmurs. The thyroid is non-enlarged & has no palpable masses. Abd exam shows epigastric tenderness w/ deep palpation but no rebound, guarding, or palpable masses. A bedside TVUS confirms the presence of an 8 week twin gestation w/ 2 normal fetal heart beats. Which of the following is the best next step in the eval of this pt? (24-urine protein collection OR urinalysis for ketones)

Urinalysis for ketones > pt has signs of hyperemesis gravidum >> labs associated w/ are: 1- *ketonuria* 2- Hypochloremic metabolic alkalosis 3- Hypokalemia 4- Hemoconcentration 5- Wt loss >5% of pre-pregnancy wt Associated w/ 1- Molar pregnancy 2- Multiple gestation 24 urine collection > only if preeclampsia is suspected

A 42F, G1P0, at 36 wks presents to the L & D unit for eval. Her feet were very swollen on awakening, & she has had difficulty ambulating. The pt's BP is 146/92. She reports good fetal mvmts w/ no contractions, vaginal bleeding, or leakage of fluid. The pt has been very tired & had had SOB. BP, 5 hrs after measurement, is 140/90 & wt is 154.3 lbs, a gain of 11 lbs in one week. PE shows pitting edema to the calves. Labs are normal. Urine dipstick shows 1+ protein & 1+ glucose. Fetal heart tracings are reactive. Which of the following is the best next step for confirming this Dx? (Observation of BP for 4 hours OR Urine protein-to-creatine ratio test)

Urine protein-to-creatinine ratio test > Pt has preeclampsia >> this should be confirmed via 1- Urine protein/creatinine ratio OR 2- 24-hrs collection for total protein *Gold standard* Observation of BP for 4 hours > pt already had elevated BP on 2 occasions >/= 4 hrs apart; no need for further monitoring >> patient has proteinuria >> need to confirm preeclampsia Dx

A 28F, gravida 2 para 1, Presents in active labor at 37 wks w/ severe abd pain & back pain. The pt had a prior CS delivery & is scheduled for a repeat CS at 39 wks. She has been normotensive throughout the pregnancy, which has been uncomplicated. Her BP is 90/60, & pulse is 120/min. PE shows a palpable, irregular protuberance in the lower abd & moderate vaginal bleeding. The cervix is 3 cm dilated & 80% effaced. A bulging bag is palpated at the cervical os, but there is not presenting fetal part. The fetal HR tracing shows a deceleration. Which of the following is the most likely Dx? (Umbilical cord prolapse OR Uterine rupture)

Uterine Rupture > Pt has a uterine rupture b/c of a complication of attempted vaginal birth after a prior CS >> *most uterine ruptures occur in pts w/ Hx of CS* May present w/: - sudden, excruciating abd pain - Bleeding may be vaginal and/or intra abdominal >> can result in *hypotension & tachy* - *Abnormal fetal HR tracing (Decels)* Distinguishing features - Loss of fetal station (no palpable fetal presenting part) - Presence of abdominally palpable fetal parts Umbilical cord prolapse > Would have a fetal presenting part w/ umbilical cord

A 34F, G3P2, has a precipitous spontaneous vaginal delivery of a 4240g (9lbs 5oz) boy at 39 wks after an uncomplicated prenatal course. Her prior pregnancies were delivered vaginally, & she has no Hx of prior surgery. After 20 mins, the placenta does not deliver. Traction on the umbilical cord & fundal massage are implemented. The umbilical cord avulses, after which the pt develops severe abd pain, SOB, & copious vaginal bleeding. BP is 70/40, pulse is 62/min. Exam shows a smooth, round, pale mass protruding from the vagina. The uterine fundus is no loner palpable at the umbilicus. Which of the following is the most likely Dx? (Uterine inversion OR Uterine rupture)

Uterine inversion > results from excessive fundal pressure & traction on the umbilical cord before placental separation >> the fundus collapses into the endometrial cavity & prolapsed thru the cervix, resulting in a *smooth, round mass protruding through vagina or cervix* > usually presents w/: 1- *Hemorrhagic shock* (pts low BP & copious bleeding) 2- *Lower Abd pain* Uterine rupture > Disruption of the wall of uterus that presents w/: 1- Severe pain 2- Antepartum bleeding 3- Loss of fetal station 4- *pt would have a Hx sig for CS delivery*

A 32F, G3P2, at 38 wks presents to the hospital w/ labor contractions. The pt's prenatal course was normal. Her second pregnancy required a CS for breech at 39 wks. BP is 100/60, pulse is 115/min, RR is 26/min. PE shows the cervix 6 cm dilated & 60% effaced w/ fetal head at 0 station. Uterine contractions are regular & occur every 4 mins, & fetal heart tracing is normal. The pt is admitted to the L & D unit for expectant MGMT & declines epi. 2 hours later, she suddenly appears restless & has intense lower abd pain. PE shows moderate vaginal bleeding, & the cervix is 6 cm dilated & 60% effaced w/ fetal head at -3 station. Fetal heart monitoring shows tachy & recurrent variable decels. Which of the following is the most likely Dx? (Fetal nuchal cord OR Uterine rupture)

Uterine rupture > typically occurs in pts w/ prior uterine surgery >> this pt had a uterine rupture at the scar of her prior CS >> often presents w/; 1- *Focal, intense abd pain* (prior to the rupture that is relieved by the rupture but resumes shorty after in diffuse distribution) 2- Hyperventilation 3- *Agitation* 4- Tachy 5- *Bleeding* (vaginal or intra-abdominal) 6- *Abnormal fetal heart tracings* (fetal tachy, recurrent decels) Fetal nuchal cord > Usually clinically insignificant >> can cause abnormal fetal heart tracings *recurrent variable decels* >> *but not significant pain or bleeding*

A 28y/o F, G1P0, at 41 wks comes to the hospital for antepartum fetal surveillance. She has had less fetal mvmts than usual but no abd pain, leakage of fluid, or vaginal bleeding. BP is 120/80. The pr's pre-pregnancy BMI was 40, and she has gained 44 lbs during the pregnancy. Her pregnancy has been otherwise uncomplicated. NST shows a baseline FHR of 140/min, moderate variability, and no accelerations for > 40 mins. Two contractions are observed on the tocodynamometer. An episode of Fetal HR decrease to 130/min began after at the peak contraction & lasted for a minute. US shows a fetus in cephalic presentation, a loop of umbilical cord near the neck, an estimated fetal wt at the 35th percentile for gestational age, and a single deepest pocket of amniotic fluid of 1cm. Biophysical profile shows: - NST. 0 - Amniotic fluid. 0 - Fetal MVMTs. 2 - Fetal tone. 2 - Fetal breathing mvmts 0 Which of the following is the most likely cause of this pts biophysical profile results? (Nuchal cord OR Uteroplacental insufficiency)

Uteroplacental insufficiency > Decreased placental function reduces the placental perfusion during contractions, which can result in intermittent fetal hypoxemia >> On a NST *intermittent hypoxemia presents as late decelerations* > often seen in *late-term pregnancies* > A BPP score of </= 4 suggests imminent risk of fetal demise d/t *uteroplacental insufficiency* Nuchal cord > are associated w/ variable decels >> *do not cause oligohydraminos or decreased fetal breathing mvmts*

A 57F, presents for eval of vaginal pruritus, pain w/ urination, & increased urinary frequency. The pt's symptoms have been persistent for several months but have intensified recently. Her husband passed away 4 yrs ago, & the pt had not been sexually active until 6 months ago w/ a new partner. She has had some pain w/ intercourse & has tried OTC water-based lubricants w/ little relief. Her last menstrual period was 7 year ago. She ha poorly controlled DM II w/ a recent Hb A1c of 8.4%. BMI is 28. Vitals are normal. Pelvic exam shows thin vulvar skin w/ reduced elasticity. The vagina appears pale, dry, & has multiple areas of petechiae. There is minimal clear discharge in the vault. Vaginal pH is 6.5. Urinalysis is normal. Which of the following is the best next step in MGMT of this pt? (Oral nitrofurantoin OR Vaginal estrogen cream)

Vaginal estrogen cream > Pt presents w/ *genitourinary syndrome of menopause*, which presents as: 1- *Vulvovaginal dryness, irritation, pruritus* 2- Dyspareunia 3- Vaginal bleeding (*petechiae*) 4- *Urinary incontinence, recurrent UTI* 5- Pelvic pressure Oral nitrofurantoin > Pt has dysuria & urinary frequency *but urinalysis is normal*

A 32F, nulliparous, starts to experience more painful contractions as she transitions into active labor. On admission, her BP is 120/80. Fetal HR tracing shows a baseline of 130/min, moderate variability, positive accelerations, & no decels. Uterine contractions occur every 2-3 mins. Cervical exam shows a 6 cm dilation, 75% effacement, and 0 station. She requests an epi for pain control. After induction of anesthesia, she begins to feel light-headed & her BP drops to 90/55, HR is 120/min, RR 12/min. The pt has normal strength & sensation of the upper extremities. What is the most probable cause of her hypotension? (Progressive hypovolemia OR Vasodilation & venous pooling) Describe the TMT for the correct answer.

Vasodilation & venous pooling > Hypotension w/ epi occurs when the sympathetic nerve fibers responsible for the vascular tone are blocked, resulting in vasodilation (venous pooling), decreased VR to right side of heart & decreased CO. TMT > can be prevented by aggressive IV fluid volume expansion prior to epi > Left uterine displacement (positioning pt on left side) to improve VR > additional IV fluid bolus > Vasopressors Progressive Hypovolemia > Would be secondary to ongoing fluid & blood loss OR Electrolyte abnormalities

A 51F, presents to the office for a routine health maintenance exam. She has no concerns today. The pt recently had a normal screening mammogram. She had DM I & her last Hb A1c was 7.6%. The pt had an abnormal Pap at age 35 that required cervical conization, but all subsequent Pap tests have been normal. She is currently sexually active & has had no pain w/ intercourse. Her last menstrual period was 2 wks ago. The pt does not drink, smoke, or use drugs. Vitals are normal. BMI is 26. PE shows depigmentation in the vuvla. What is the most likely Dx? (Vitiligo OR Lichen planus)

Vitiligo > Although vitiligo has a more generalized distribution, a subset of pts have lesions that affect only the genital or oral mucosa Lichen planus > Pt would have bright, glazed, red-purple plaques & papules w/ an overlying white, lacy pattern (Wickham striae) > pt would have significant symptoms, such as: 1- Pruritus 2- Pain 3- Dyspareunia >> but *no associated hypopigmentation*

A 70F, presents d/t vulvar pruritus. The pt had had increasing pruritus over the past 4 months that has not improved w/ topical emollients. She has no pelvic pain, vaginal bleeding, or abnormal vaginal discharge. The pt had similar symptoms 20 yrs ago, for which she was prescribed a corticosteroid cream that resolved the symptoms. She has not been sexually active for the last 10 years. The pt has hypercholesterolemia & DM II. Her last Pap test w/ HPV contesting was normal 5 yrs ago. BP is 132/78 & pulse is 84/min. BMI is 32. On pelvic exam, there is firm, white plaque w/ overlying excoriations on the left album majus. Mild atrophy of the bilateral labia minora is noted, and the vagina has minimal rugation but no lesions. Which of the following is the best next step in mgmt? (High-dose topical corticosteroids OR Vulvar biopsy)

Vulvar biopsy > Pt has *unifocal, firm, white vulvar plaque* which is concerning for *squamous cell carcinoma* >> RF is chronic lichen sclerosis (pt continued to have it even after application of cream) >> this is concerning for malignant transformation >> next best step is biopsy High-dose topical corticosteroids > If this pts lesions is benign on biopsy and is recurrence of lichen sclerosis >> she can be treated w/ this

A 52F, postmenopausal presents for an eval of vulvar irritation. The pt wears sanitary napkins d/t stress incontinence & occasionally notices streaked of blood on the napkin. She is sexually active & has had some pain w/ sexual intercourse. The pt underwent a hysterectomy & BSO at age 48 for adenomyosis. She has had abnormal Pap tests previously, but normal colposcopy results. The pt currently uses a nicotine patch for smoking cessation. Vitals are normal. PE shows multiple vulvar excoriations w/ surrounding erythema. There is an erythematous, friable plaque on the left labium majus. On speculum, the vagina has no lesions or abnormal discharge. Which of the following is the most likely Dx? (Lichen planus OR Lichen sclerosis OR Vulvar cancer)

Vulvar cancer > Presents w/: 1- *Erythematous, friable plaque on labium majus that produces persistent vulvar irritation and/or pain* 2- May also have *intermittent bleeding & dyspareunia* RF 1- *Hx of HPV infection* 2- Chronic tobacco use (can't clear it) 3- Immunodeficiency (can't clear it) Lichen sclerosis > Presents w/ multiple white papules that converge into plaques *single lesion is uncommon* Lichen Planus > Typically presents w/ Pruritic, purple hued plaques that are sometimes associated w/ thin, white striae around the labia and vulva *Wickham striae*

A 34F, G1P1, presents for eval of her contraceptive method. For the past 8 months, the pt has experienced unprecedented bleeding since having a subnormal progestin implant placed & requests removal. She previously used combined oral contraceptives & would like to return to that method. The pt has primary HTN that was Dx'd last year & has been well controlled w/ Hydrochlorothiazide. She takes no meds & has no allergies. Her father & brother have DM II. The pt does not drink, smoke, or use drugs. PE is normal. BMI is 24. BP is 130/75. Combo oral contraception increases the pt's risk of which of the following? (Endometrial cancer OR Worsening HTN)

Worsening HTN > Oral contraceptives increase risk for: 1- Venous thromboembolism 2- HTN 3- Hepatic adenoma 4- Stroke, MI 5- Cervical cancer Endometrial cancer - Combined oral contraceptives are *protective for endometrial cancer* >> D/t progestin which counteracts the estrogen effects on endometrium


Ensembles d'études connexes

5 - TCP/IP Protocols - Self Test

View Set

BSC2011 Exam 3 learning catalytics

View Set

Hinduism and Buddhism & India's Caste System

View Set

CH1 Completing the Application, Underwriting and Delivering the Policy

View Set

ITN 260 John Tyler Network Security Basics Chapter 5 Quiz

View Set